SSC CGL Tier-I 07 June 2019 Shift-III Previous Year Paper

SSC CGL

(7 June 2019 Shift-III) 

Reasoning 

Instructions For the following questions answer them individually 

Q.1 Select the option that is related to the fifth term in the same way as the second term is related to the first term and the fourth term is related to the third term. 

6472 : 25 :: 7343 : 41 :: 6582 : ? 

A 14 

B 16 

C 18 

D 13 

Answer: B 

 

Q.2 If + denotes − − denotes ∗ ∗ denotes / / denotes + then what will be the numeric value of 60 ∗ 10/40 + 6 − 5 =? 

A 16 

B 200 

C 3 

D 144 

Answer: A 

 

Q.3 Three of the following four letter-clusters are alike in a certain way and one is different. Pick the odd one out. 

A FNMG 

B SKLV 

C PXWQ 

D BKJC 

Answer: B 

 

Q.4 How many triangles are there in the following figure ? 

A 29 

B 27 

C 25 

D 31 

Answer: A 

 

Q.5 Arrange the following words in a logical and meaningful order. 

1. Doctor 2. Recovery 3: Hospital 4. Home 5: Sickness 6. Medicine store 

A 1, 5, 3, 6, 4, 2 

B 4, 5, 6, 1, 3, 2 

C 5, 3, 6, 1, 3, 2 

D 5, 3, 1, 6, 2, 4 

Answer: D 

 

Q.6 Three of the following four words are alike in a certain way and one is different. Pick the odd word out. 

A Afghanistan 

B Nepal 

C Indonesia 

D Ireland 

Answer: D 

 

Q.7 Two different positions of the same dice are shown. Which number will be at the top if 3 is at the bottom ? 

A 5 

B 1 

C 4 

D 2 

Answer: A 

 

Q.8 Acycle was sold for ₹4,140 with a profit of 15%. At what price could it have been sold to earn a profit of 25% ? 

A ₹4,200 

B ₹4,500 

C ₹4,850 

D ₹4,350 

Answer: B 

 

Q.9 Which number will replace the question mark (?) in the following series? 

7, 11, 18, 29, ?, 76 

A 47 

B 53 

C 61 

D 44 

Answer: A 

 

Q.10 Two statements are given, followed by three conclusions numbered I, II and III. Assuming the statements to be true even if they seem to be at variance with commonly known facts, decide which of the conclusions logically follow(s) from the statements. Statements: 

Some quadrilaterals are squares. 

All squares are rhombuses. 

Conclusions: 

I. No quadrilateral is a rhombus. 

II. All rhombuses are squares. 

III. Some quadrilaterals are rhombuses. 

A Only conclusion I follows 

B Only conclusion II follows 

C None of the conclusions follows 

D Only conclusion III follows 

Answer: D 

 

Q.11 Three of the following four word pairs are alike in a certain way and one is different. Pick the odd word out. 

A Tamil Nadu : South 

B Punjab : North 

C Assam : North-East 

D Kerala : North-West 

Answer: D 

 

Q.12 Select the option that is related to the third letter-cluster in the same way as the second letter-cluster is related to the first letter-cluster. FLQV : JISU :: DINR : ? 

A HGPQ 

B HMPS 

C HGLQ 

D FGQN 

Answer: A 

 

Q.13 Three of the following four letter-clusters are alike in a certain way and one is different. Pick the odd one out. 

A LPQ 

B DHL 

C HLP 

D PTX 

Answer: A 

 

Q.14 Select the set in which the numbers are related in the same way as are the numbers of the following set. (11, 16, 20) 

A (21, 26, 32) 

B (15, 20, 24) 

C (9, 14, 19) 

D (17, 23, 26) 

Answer: B 

 

Q.15 Select the number pair in which the two numbers are related in the same way as are the two numbers of the following number-pair. 27 : 8 

A 19 : 5 

B 12 : 3 

C 33 : 9 

D 39 : 14 

Answer: B 

 

Q.16 A square paper is folded and cut as shown below. How will it appear when unfolded ? 

Answer: B 

 

Q.17 If the word ANGEL is coded as BOHFM, then what will be the third alphabet in the code for the word SAVAGE ? 

A X 

B B 

C W 

D U 

Answer: C 

 

Q.18 Three of the following four numbers are alike in a certain way and one is different. Pick the number that is different from the rest. 

A 2197 

B 343 

C 731 

D 1331 

Answer: C 

 

Q.19 Select the combination of letters that when sequentially placed in the gaps of the given letter series will complete the series. cd_ab_cd_abb_dda_b 

A dbdcb 

B bbcdb 

C dbcbc 

D cbdab 

Answer: A

 

Q.20 Select the mirror image of the given figure when the mirror is placed to the right of the figure. 

Answer: A 

 

Q.21 Select the figure that will come next in the following figure series. 

Answer: D 

 


Q.22 Select the word-pair in which the two words are related in the same wayas are the two words in the following word pair. Mnemonic : Memory 

A Sedative : Sleep 

B Drama : Acting 

C Audition : Music 

D Audience : Speech 

Answer: A 

 

Q.23 ‘Judgment’ is related to ‘Deliver’ in the same way as ‘Rule’ is related to ‘………’ 

A Follow 

B Practice 

C Render 

D Implement 

Answer: D 

 

Q.24 Select the Venn diagram that best illustrates the relationship between the following classes. Villagers, Poor Persons, Males 

Answer: C 

 

Q.25 Select the figure in which the given figure is embedded. 

Answer: D 

 

General knowledge 

Instructions For the following questions answer them individually 

Q.26 The Almatti dam project on the Krishna river was an issue between which states? 

A Karnataka and Tamil Nadu 

B Andhra Pradesh and Tamil Nadu 

C Karnataka and Andhra Pradesh 

D Karnataka and Goa 

Answer: C 

 

Q.27 In March 2018, Indian actress ………… was namedas the new ambassadorof the international award- winning non-profit organisation, Educate Girls. 

A Aishwarya Rai 

B Deepika Padukone 

C Priyanka Chopra 

D Katrina Kaif 

Answer: D 

 

Q.28 The maximum number of nominated members to Lok Sabha is ………… 

A 4 

B 2 

C 1 

D 3 

Answer: B 

 

Q.29 During whose reign did the Chinese traveller Hiuen Tsang visit India? 

A Chandragupta I 

B Harshavardhana 

C Samudragupta 

D Chandragupta Vikramaditya 

Answer: B 

 

Q.30 The base financial year for the calculation of the all India Index of Industrial Production (IIP) is: 

A 2005-2006 

B 2004-2005 

C 2011-2012 

D 2010-2011 

Answer: C 

 

Q.31 What does the Lorenz Curve indicate? 

A Relationship between the price of a certain commodity and its demand 

B Rate of employment 

C Taxable income elasticity 

D Income distribution 

Answer: D 

 

Q.32 The Suri king ………. was defeated by Humayun to regain his kingdom. 

A Mahmood Suri 

B Sher Shah Suri 

C Bahalol Suri 

D Sikandar Suri 

Answer: D

 

Q.33 ……….. built the world famous Harmandar Sahib, popularly known as the Golden Temple in Amritsar. 

A Guru Arjan Dev 

B Guru Siri Har Rai 

C Guru Angad Dev 

D Guru Ram Das 

Answer: A 

 

Q.34 ‘Shifting cultivation’ is also known as ………. in north-east India. 

A Chena 

B Ladang 

C Jhum 

D Logan 

Answer: C 

 

Q.35 Who decides on the issue related to the disqualification of a Member of Lok Sabha under Tenth Schedule? 

A President 

B Speaker 

C Prime Minister 

D Vice President 

Answer: B 

 

Q.36 In March 2019, the Government of …….. launched the ‘Lose to Win’ programme to assist overweight employees shed extra kilos and adopt a healthy lifestyle. 

A Australia 

B UK 

C UAE 

D US 

Answer: C 

 

Q.37 …….. is famous for outstanding specimens of Buddhistart and architecture, belonging to the period between the 3rd century B.C. and the 12th century A.D. 

A Satna 

B Vidisha 

C Dewas 

D Sanchi 

Answer: D 

 

Q.38 The Qutub Minar was named after the Sufi saint ………. 

A Alauddin Sabir Kaliyari 

B Qutub-ud-Din Aibak 

C Syed Waheed Ashraf 

D Khwaja Qutbuddin Bakhtiyar Kaki 

Answer: D 

 

Q.39 ……… was the first person to isolate methane gas. He discovered that methane mixed with air could be exploded using an electric spark. 

A Louis Pasteur 

B William Thomson 

C William Crookes 

D Alessandro Volta 

Answer: D 

 

Q.40 ……….. is an important road link between Srinagar on one side and Kargil and Leh on the other side. 

A Zoji La 

B Muling La 

C Qara Tag La 

D Shipki La 

Answer: A 

 

Q.41 Who invented the atomic battery in 1912 ? 

A Benjamin Franklin 

B Henry Moseley 

C Alessandro Volta 

D Louis Pasteur 

Answer: B 

 

Q.42 Asian Games, also knownasAsiad, is a multi-sport event held every ……….. years among athletes from all over Asia. 

A six 

B five 

C four 

D three 

Answer: C 

 

Q.43 ………. won gold in the Men’s Freestyle wrestling 65 kg category at the Dan Kolov 2019 wrestling event. 

A Bajrang Punia 

B Yogeshwar Dutt 

C Sandeep Tomar 

D Sushil Kumar 

Answer: A 

 

Q.44 ………. codified the first two laws of thermodynamics and deduced that the absolute zero of temperature is −273.15∘ C. He was honoured for this with the naming of the Kelvin temperature scale. 

A Robert Hooke 

B William Crookes 

C William Thomson 

D Luis Alvarez 

Answer: C 

 

Q.45 The theory that dinosaurs were driven to extinction by the after math of a large asteroid impact on Earth was given by ………. 

A Wilhelm Réntgen 

B Luis Alvarez 

C Henry Moseley 

D William Crookes 

Answer: B 

 

Q.46 Who was the fifth of the ten Sikh gurus? 

A Guru Ramdas 

B Guru Angad 

C Guru Arjan Dev 

D Guru Hargobind 

Answer: C 

 

Q.47 The deficiency of which nutrient causes night blindness ? 

A Vitamin A 

B Vitamin C 

C Proteins 

D Vitamin K 

Answer: A 

 

Q.48 Flying Officer ……… of the Indian Air Force (IAF) created history by becoming the first Indian woman to fly her first solo fighter flight in a Russian made MiG-21 fighter. 

A Mohana Singh 

B Avani Chaturvedi 

C Bhawana Kanth 

D Anjali Gupta 

Answer: B 

 

Q.49 Brazilian physicist and astronomer was awarded the 2019 Templeton Prize, worth $1.4 million, for his work blending science and spirituality. 

A Charles Messier 

B Marcelo Gleiser 

C Nicolaus Copernicus 

D Edwin Hubble 

Answer: B 

 

Q.50 In March 2019, a science teacher from rural Kenya named ……… won the prestigious Global Teacher Prize 2019, worth $1 million, which honours the world’s best teacher. 

A Andria Zafirakou 

B Peter Tabichi 

C Nancie Atwell 

D Hanan Al Hroub 

Answer: B 

 

Quantitative Aptitude

Instructions For the following questions answer them individually 

Q.51 The value of 6 − 6 ÷ 6 × 6 + (6 ÷ 6 of 6) × 6 − ( 3⅔ ÷ 11/30 of  ⅔) ÷ 5 is: 

A 0 

B -1 

C -2 

D 2 

Answer: C 

 

Q.52 If 6x /(2x2 +5x−2) = 1, x > 0,then the value of x3 + 1/x3 is: 

A ⅜√17

B ⅝√17

C ¾√17

D ⁵/₁₆√17

Answer: B 

 

Q.53 Two chords AB and CD of lengths 5 cm and 11 cm respectively are parallel and are on the same side of the centre O of a circle. If the distance between the chords is 3 cm, then what is the diameter of the circle? 

A √146 cm 

B √142 cm 

C 37 cm 

D 38 cm 

Answer: A 

 

Q.54 The table shows the production of different types of cars (in thousands). 

The average production of cars in 2018 is approximately what percent less than the total production of type D cars in 2015 and type B cars in 2017 taken together ? 

A 44.2% 

B 43.6% 

C 45.8% 

D 42.4% 

Answer: C 

 

Q.55 If 1/(cosec θ−1)+1/(cosec θ+1)= 2secθ,0 < θ < 90, then the value of (cotθ + cosθ) is: 

A (2-√3)/√2

B (2-√2)/√2

C (1-√2)/√2

D 1+√2

Answer: B 

 

Q.56 2tan30/(1−tan2 30)= 

A √3

B 1/√3

C ⅓

D 3 

Answer: A

 

Q.57 A journey of 96 km takes one hour less by a fast train (A) than by a slow train (B). If the average speed of B is 16 km/h less than that of A, then the average speed (in km/h) of A is: 

A 60 

B 54 

C 64 

D 48 

Answer: D 

 

Q.58 The value of sin2 30 cos2 45 + 4tan2 30 + ½sin2 90 + 2cos90is: 

A 2 

B ⁴⁷/₂₄

C ¹⁵/₈

D ²³/₁₂

Answer: B 

 

Q.59 What is the compound interest on a sum of ₹8,100 for 1¼ years at 8% per annum,if the interest is compounded 5-monthly? (Nearest to ₹1) 

A ₹837 

B ₹873 

C ₹824 

D ₹842 

Answer: A 

 

Q.60 In △ABC, AM BC and AN is the bisector of ∠A. What is the measure of ∠MAN, if ∠B =55, and ∠C = 35

A 12

B 5

C 10

D 15

Answer: C 

 

Q.61 A cylindrical road roller made of metal is one meter long.Its inner radius is 27 cm and the thickness of the metal sheet rolled into it is 9 cm. What is the weight of the roller, if 1 cm3 of the metal weighs 8 g ? 

A 453.6πkg 

B 449πkg 

C 441πkg 

D 442.4πkg 

Answer: A 

 

Q.62 A person sells an article at a profit of 12%. If he had purchased it for 12% less and sold it for ₹9 less, he would have gained 27%. What is the original cost price of the article? 

A ₹3,750 

B ₹4,500 

C ₹4,250 

D ₹4,000 

Answer: A 

 

Q.63 The table shows the production of different types of cars (in thousands). 

What is the ratio of the total production of type A cars in 2017 and type C cars in 2014 taken together to the total production of type B cars in 2014, type C cars in 2017 and type E cars in 2018 taken together? 

A 3 : 4 

B 2 : 3 

C 12 : 11 

D 5 : 6 

Answer: A 

 

Q.64 If 10-digit number 67127y76x2 is divisible by 88, then the value of (7x- 2y)is: 

A 10 

B 7 

C 5 

D 3 

Answer: B 

 

Q.65 Surbhi spends 75% of her income. If her income increases by 20% and savings decrease by 1%, then the percentage increase in her expenditure is: 

A 22 

B 2.7 

C 2.2 

D 27 

Answer: D 

 

Q.66 The ratio of the efficiencies of A, B and C is 3 : 5 : 1. Working together, they can complete a piece of work in 5 days. A and B work together for 3 days. The remaining work will be completed by C alone in? 

A 24 days 

B 18 days 

C 15 days 

D 21 days 

Answer: D 

 

Q.67 In a circle with centre O, PQR is a tangent at the point Q on it. AB is a chord in the circle parallel to the tangent such that ∠BQR = 70. What is the measure of ∠AQB ? 

A 60

B 35

C 55

D 40

Answer: D 

 

Q.68 The table shows the production of different types of cars (in thousands).

If the data related to the production of type D cars is represented by a pie-chart. then the central angle of the sector representing the production of cars in 2017 will be: 

A 81.6 

B 50∘ 

C 75.6 

D 78 

Answer: D 

 

Q.69 Four different positive numbers are written in ascending order. One-third of the average of all the four numbers is 19 less than the greatest of these numbers. If the average of the first three numbers is 12, the greatest number among the given numbers is: 

A 21 

B 24 

C 22 

D 25 

Answer: B 

 

Q.70 If tanθ/(1−cotθ) + cotθ/(1−tanθ) = 1 + k , then k =? 

A cotθ + secθ 

B tanθ + secθ 

C tanθ cosecθ 

D cosecθ secθ 

Answer: D 

 

Q.71 The table shows the production of different types of cars (in thousands). 

The total production of type A cars in 2016 and type E cars in 2014 taken together is what percent of the total production of type C cars during 2014 to 2018 ? 

A 32 

B 40 

C 35 

D 36 

Answer: D 

 

Q.72 If (8x3 + 27y3 ) ÷ (2x + 3y) = (Ax2 + Bxy + Cy2 ), then the value of (5A + 4B + 3C) is: 

A 26 

B 27 

C 24 

D 23 

Answer: D 

 

Q.73 The base and altitude of an isosceles triangle are 10 cm and 12 cm respectively. Then the length of each equal side is: 

A 13 cm 

B 7.5 cm 

C 10 cm 

D 8.5 cm 

Answer: A 

 

Q.74 By how much above the cost price should an article be marked up for sale so that after allowing two successive discounts of 20% and 6.25% onit, a net gain of 20% is made on the cost? 

A 50% 

B 60% 

C 66 2 3% 

D 4641 % 

Answer: B 

 

Q.75 Asum of ₹x is divided among A, B and C such that the ratio of shares of A and B is 7 : 12 and that of B and C is 8: S. If the difference in the shares of A and C is ₹214, then the value of x is: 

A 11,342 

B 11,128 

C 11,556 

D 11,770 

Answer: A 

 

English 

Instructions For the following questions answer them individually 

Q.76 Select one word substitution for the given words. A trade that is prohibited by law 

A illicit 

B inapt 

C illusive 

D incredible 

Answer: A 

 

Q.77 Select the correctly spelt word. 

A concious 

B contemptible 

C complascent 

D corigible 

Answer: B 

 

Q.78 Select the most appropriate option to substitute the underlined segment in the given sentence. If no substitution is required, select No improvement. We must endeavour to increase women’s access for education also employment. 

A No improvement 

B access to education and employment 

C excess to education or employment 

D access in education also employment 

Answer: B 

 

Q.79 Select the most appropriate meaning of the underlined idiom in the given sentence. He kicked up a row when he was denied entry to the amusement park. 

A waited in a queue 

B kicked the guard 

C cried with sorrow 

D created a fuss 

Answer: D 

 

Q.80 Select the correctly spelt word. 

A apearance 

B arrogence 

C attendence 

D abundance 

Answer: D 

 

Q.81 Fill in the blank with the most appropriate word. The committee members were in ………. over the budget for the function. 

A dissent 

B deference 

C difference 

D descent 

Answer: A 

 

Instructions: In the following passage some words have been deleted. Fill in the blanks with the help of the alternatives given. Select the most appropriate option for each blank. 

Comprehension: There was still a light on in Mo’s room. He often stayed (1)……… reading late into the night. Meggie had (2)………. her love of books from him. When she (3)………. refuge with him from a bad dream, nothing could lull (4)……….. back to sleep better than Mo’s calm breathing (5)………… her and the sound of the pages turning. 

Q.82 Select the most appropriate option to fill in blank No.1. 

A out 

B up 

C for 

D away 

Answer: B 

 

Q.83 Select the most appropriate option to fill in blank No.2. 

A inherited 

B gained 

C extracted 

D procured 

Answer: A 

 

Q.84 Select the most appropriate option to fill in blank No.3. 

A has taken 

B took 

C takes 

D will take 

Answer: B 

 

Q.85 Select the most appropriate option to fill in blank No.4. 

A it 

B her 

C them 

D his 

Answer: B 

 

Q.86 Select the most appropriate option to fill in blank No.5. 

A behind 

B beside 

C against 

D around 

Answer: B 

 

Instructions For the following questions answer them individually 

Q.87 In the sentence identify the segment which contains the grammatical error. Each of the girls have given an impressive dance performance. 

A an impressive 

B have given 

C dance performance 

D Each of the girls 

Answer: B 

 

Q.88 Select the most appropriate option to substitute the underlined segment in the given sentence. If no substitution is required, select No improvement. Let us not neglect important aspects of life although pursue momentary pleasures. 

A by pursued moment pleasures. 

B No improvement 

C while pursuing momentary pleasures. 

D through pursued in momentary pleasures. 

Answer: C 

 

Q.89 Given below are four jumbled sentences. Select the option that gives their correct order. 

A. Only the fittest creatures can survive while competing for food. 

B. Why do some species survive and others become extinct? 

C. His answer was that there is ceaseless struggle for life among all creatures. 

D. This was the question that Darwin asked himself. 

A BCDA 

B ADCB 

C BDCA 

D ABDC 

Answer: C 

 

Q.90 Select the most appropriate antonym of the given word. TRANSIENT 

A permanent 

B temporal 

C celestial 

D stationary 

Answer: A 

 

Q.91 Select the most appropriate meaning of the underlined idiom in the given sentence. The heavy losses in business came like a bolt from the blue. 

A a windfall 

B an ominous warning 

C an unexpected disaster 

D a thunderstorm 

Answer: C 

 

Q.92 Select the most appropriate synonym of the given word. RETICENT 

A extrovert 

B garrulous 

C confident 

D silent 

Answer: D 

 

Q.93 Select the correct passive form of the given sentence. Some People believe that discipline means blind submission to authority. 

A It has been believed by some people that discipline means blind submission to authority. 

B It was believed by some people that discipline meant blind submission to authority. 

C It is believed by some people that discipline means blind submission to authority. 

D It is belief by some people that discipline means blind submission to authority. 

Answer: C 

 

Q.94 Select the correct active form of the given sentence. The hunchback was being laughed at by everyone. 

A Everyone laughs at the hunchback. 

B Everyone was laughing at the hunchback. 

C Everyone is laughing at the hunchback. 

D Everyone laughed at the hunchback. 

Answer: B

 

Q.95 Given below are four jumbled sentences. Select the option that gives their correct order. 

A. What can we do to avoid anger and provocation? 

B. Anger and tension are prevalent due to conflicts arising out of these differences. 

C. The society is full of people who think and act differently. 

D. We need to develop in ourselves the capacity for conversion to turn negative experiences into positive thinking. 

A ABDC 

B CBAD 

C ACBD 

D CADB 

Answer: B 

 

Q.96 Select the most appropriate synonym of the given word. FURY 

A fright 

B sorrow 

C cruelty 

D anger 

Answer: D 

 

Q.97 Fill in the blank with the most appropriate word. The old man wished to donate his ……….. wealth for the upliftment of the downtrodden. 

A immense 

B elusive 

C intense 

D eminent 

Answer: A 

 

Q.98 In the sentence identify the segment which contains the grammatical error. If so many catches had not been dropped, we would have won the match. 

A had not being dropped 

B won the match 

C If so many catches 

D we would have 

Answer: A 

 

Q.99 Select one word substitution for the given words. To give up the throne 

A usurp 

B abdicate 

C consign 

D bequeath 

Answer: B 

 

Q.100 Select the most appropriate antonym of the given word. LIBERTY 

A freedom 

B slavery 

C reservation 

D autonomy 

Answer: B 

SSC CGL Tier-I 07 June 2019 Shift-II Previous Year Paper

SSC CGL

(7 June 2019 Shift-II) 

Reasoning 

Instructions For the following questions answer them individually 

Q.1 A square paper is folded and cut as shown below. How will it appear when unfolded ? 

Answer: C 

Q.2 

Select the figure that will come next in the following figure series. 

Answer: B 

Q.3 Which number will replace the question mark (?) in the following series? 5.12.26, 54, ? 

A 112 

B 92 

C 110 

D 98 

Answer: D 

Q.4 Select the set in which the numbers are related in the same way as are the numbers of the following set. (8, 15, 9) 

A (5, 17, 11) 

B (4, 16, 12) 

C (7, 13, 14) 

D (9, 12, 15) 

Answer: B 

Q.5 The ratio of the present age of Mitali to that of Shabnam is 4 : 7. If the difference between the present ages of Shabnam and Mitali after 5 years will be 13 years, then what is the sum of the present ages of Mitali and Shabnam? 

A 74 years 

B 60 years 

C 66 years 

D 64 years 

Answer: C 

Q.6 P is the sister of Q and mother of R. S is the daughter-in-law of Wand wife of Q. W’s grandson M is the nephew of P and brother of V. W has only two children. How is Q related to V ? 

A Uncle 

B Father 

C Brother 

D Son 

Answer: B 

Q.7 Select the word-pair in which the two words are related in the same way as are the two words in the following word pair. Caress : Affection 

A Crime : Confession 

B Interview : Selection 

C Anger : Emotion 

D Kick : Hostility 

Answer: D 

Q.8 If CLEAN is coded as 1415123 and BLOW is coded as 2315122, then how will JOKE be coded as ? 

A 4111612 

B 5121611 

C 5111510 

D 1011155 

Answer: C 

Q.9 How many triangles are there in the following figure ? 

A 18 

B 20 

C 16 

D 14 

Answer: A 

Q.10 Select the mirror image of the given figure when the mirror is placed to the right of the figure. 

Answer: B 

Q.11 Select the option that is related to the third letter-cluster in the same way as the second letter-cluster is related to the first letter-cluster. DFIM : WURN :: GENP : ? 

A SOLJ 

B TROK 

C TPNL 

D TPMK 

Answer: D 

Q.12 Select the number-pair in which the two numbers are related in the same way as are the two numbers of the following number-pair. 12 : 42 

A 15 : 52 

B 24 : 82 

C 8 : 28 

D 16 : 53 

Answer: C 

Q.13 Two different 16:53 positions of the same dice are shown. Which number will be at the top if 6 is at the bottom ?

A 5 

B 3 

C 4 

D 2 

Answer: D 

Q.14 Select the set in which the numbers are related in the same way as are the numbers of the following set. (3, 16, 5) 

A (9,35,11) 

B (7,32,9) 

C (11,45,14) 

D (5,26,9) 

Answer: B 

Q.15 Two statements are given, followed by three conclusions numbered I, II and III. Assuming the statements to be true even if they seem to be at variance with commonly known facts, decide which of the conclusions logically follow(s) from the statements. Statements: Some roads are streets. Some streets are lanes. Conclusions: I. No road is a lane. II. All streets are lanes. III. Some lanes are roads. 

A Only conclusion II follows 

B Only conclusionsII andIII follow 

C Either conclusions I or ITI follows 

D Only conclusion I follows 

Answer: C 

Q.16 In a code language, REGULAR is written as GERTRAL. How will BROTHER be written as in that language? 

A ORBSREH 

B ROBSERH 

C ORBUREH 

D ORBEHTS 

Answer: A 

Q.17 Select the Venn diagram that best illustrates the relationship between the following classes. Graduates, Teachers, Literates 

Answer: A 

Q.18 Select the figure in which the given figure is embedded. 

Answer: D 

Q.19 Three of the following four numbers are alike in a certain way and one is different. Pick the number that is different from the rest. 

A 39 

B 57 

C 43 

D 63 

Answer: C

Q.20 Arrange the following words in a logical and meaningful order. 1. Learn 2. College 3: Degree 4. Admission 5. Assessment 6. Class 

A 4, 2, 1, 6, 3, 5 

B 2, 1, 4, 5, 6, 3 

C 4, 2, 6, 1, 5, 3 

D 5, 4, 2, 1, 6, 3 

Answer: C 

Q.21 “division” is to “unity” as “repair” is to_____ 

A rough 

B damage 

C inconvenience 

D stitch 

Answer: B 

Q.22 Which two signs should be interchanged in the following equation to make it correct? 

9 − 3 + 12 × 8 ÷ 4 = 11 

A + and − 

B + and × 

C − and ÷ 

D + and ÷ 

Answer: D 

Q.23 Three of the following four letter-clusters are alike in a certain way and one is different. Pick the odd one out. 

A RSTY 

B HJLO 

C BDFA 

D PQRU 

Answer: A 

Q.24 Three of the following four words are alike in a certain way and one is different. Pick the odd word out. 

A Condemn 

B Disparage 

C Denigrate 

D Awkward 

Answer: D 

Q.25 Select the combination of letters that when sequentially placed in the gaps of the given letter series will complete the series. db_cbd_ba_bd_bac_d 

A adbcd 

B adcdb 

C bdcbd 

D dacdb 

Answer: B 

General knowledge 

Instructions For the following questions answer them individually 

Q.26 On March 23rd, India observes _______ day/divas as a tribute to Bhagat Singh, Sukhdev and Rajguru on their death anniversary. 

A Vidroh Divas 

B Shaheed Divas 

C Protest Day 

D Tribute Day 

Answer: B 

Q.27 Which of the following is the longest rivers in India? 

A Kaveri 

B Godavari 

C Brahmaputra 

D Ganga 

Answer: C 

Q.28 Akbar was succeeded by his son, Salim, who took the title of_______, meaning ‘Conqueror of the World’. 

A Shah Jahan 

B Jahangir 

C Badshah 

D Jahapana 

Answer: B 

Q.29 Who was the prime minister of India during ”the Emergency” between the year 1975 to 1977? 

A Indira Gandhi 

B Rajiv Gandhi 

C Moraj Desai 

D Charan Singh 

Answer: A 

Q.30 The Victoria Memorial, conceived by Lord Curzon, represents the architectural climax of _________ city. 

A Jaipur 

B Kolkata 

C Mumbai 

D Delhi 

Answer: B 

Q.31 World _________Day 2019 was observed on 23 March with the theme ‘The Sun, the Earth and the weather’. 

A Geographical 

B Geological 

C Astrological 

D Meteorological 

Answer: D 

Q.32 Indian naval officer Commander _____ became the only Asian to participate in the prestigious Golden Globe Race (GGR), which commenced from Les Sables d’Olonne harbour in France on 1 July 2018 

A Pradeep Singh 

B Mahendra Nath Mulla 

C Abhilash Tomy 

D Rajesh Dhankhar 

Answer: C

Q.33 Pierre Curie shared the 1903 Nobel Prize in Physics with his wife, Marie Curie, and Henri Becquerel for their discoveries related to _______. 

A thermodynamics 

B mirror images 

C radioactivity 

D X-rays 

Answer: C 

Q.34 A ____________ occurs when a government’s total expenditures exceed the revenue that it generates, excluding money from borrowings 

A Current Account Deficit 

B Budgetary Deficit 

C Revenue Deficit 

D Fiscal Deficit 

Answer: D 

Q.35 Who won the Nobel Prize in 1906 for his neuron doctrine? 

A Pierre Curie 

B Santiago Ramón y Cajal 

C Henry Moseley 

D Luis Alvarez 

Answer: B 

Q.36 Which of the following was the most important characteristic of India’s trade throughout the colonial period? 

A Import surplus 

B Import deficiency 

C Export deficiency 

D Export surplus 

Answer: D 

Q.37 William Crookes was a physical chemist who discovered and named the element _______ 

A plutonium 

B germanium 

C thallium 

D beryllium 

Answer: C 

Q.38 Justice_____ , who retired from the Supreme Court of India on 6 July 2018, has been appointed as the new chairperson of the National Green Tribunal 

A Justice Jawad Rahim 

B Adarsh Kumar Goel 

C Raghuvendra Singh 

D Sonam Phintso Wangdi 

Answer: B 

Q.39 Which Indian monument was built by Maharaja Sawai Pratap Singh in the year 1799? 

A Hawa Mahal 

B Leh Palace 

C Mattancherry Palace 

D Mysore Palace 

Answer: A 

Q.40 The Red Fort and the Jama Masjid in Delhi stand out as towering achievements of architecture during the reign of_______ 

A Shah Jahan 

B Akbar 

C Jehangir 

D Aurangzeb 

Answer: A 

Q.41 ___________ won a silver medal at the GeeBee Boxing Tournament which was held at Helsinki, Finland 

A Naveen Kumar 

B Sachin Siwach 

C Sumit Sangwan 

D Mohammed Hussamuddin 

Answer: D

Q.42 Name the longest river in Asia 

A Yenisei 

B Amazon 

C Nile 

D Yangtze 

38th

Answer: D 

Q.43 Electron was discovered in the year 1897 by _______ 

A T. A. Edison 

B Nicola Tesla 

C J J Thomson 

D Isaac Newton 

Answer: C 

Q.44 Humayun’s heir,_____, was born in exile and was only 13 years old when his father died. 

A Akbar 

B Shah Jahan 

C Jahangir 

D Babur 

Answer: A 

Q.45 Who had built Taj Mahal, for his wife Mumtaz Mahal along the banks of the Yamuna River in Agra? 

A Aurangzeb 

B Akbar 

C Jahangir 

D Shah Jahan 

Answer: D 

Q.46 Which river passes through maximum number of countries? 

A Rhine 

B Danube 

C Amazon 

D Volga 

Answer: B 

Q.47 In its Global Economic Outlook report, Fitch Ratings cut India’s economic growth forecast for the next financial year 2019-20 starting from April 1 to ___% from its previous estimate of 7%. 

A 6.2 

B 5.4 

C 6.8 

D 6.1 

Answer: C 

Q.48 _______ Constition Amendment Act, 2002 provided for free and compulsory elementary education to all children 

A 84th 

B 85th 

C 86th 

D 87th 

Answer: C 

Q.49 ________ received the very first Nobel Prize in Physics in 1901 for his discovery of X-rays 

A Wilhelm Röntgen 

B William Thomson 

C Louis Pasteur 

D William Crookes 

Answer: A 

Q.50 There were three bronze medal lists from India at the 38th GeeBee Boxing Tournament which was held at Helsinki, Finland. Who amongst the following is NOT one of them? 

A Naveen Kumar 

B Dinesh Dagar 

C Sumit Sangwan 

D Sachin Siwach 

Answer: B 

Quant 

Instructions For the following questions answer them individually 

Q.51 A sum of ₹x was borrowed and paid back in two equal yearly instalments, each of ₹35,280. If the rate of interest was 5%, compounded annually, then the value of x is: 

A 64,400 

B 65,600 

C 64,800 

D 65,400 

Answer: B 

Q.52 In △ABC, ∠A is a right angle. The lengths of AC and BC are 6 cm and 10 cm respectively. Point D is on AB such that BD = 4 cm. What is the length of CD? 

A 2√13cm 

B 3√10cm 

C 2√10cm 

D 3√13cm 

Answer: A 

Q.53 ABCD is a quadrilateral whose side AB is the diameter of a circle through A, B, C and D. If ∠ADC =130 , then the measure of ∠BAC is: 

A 40

B 45

C 35

D 50

Answer: A 

Q.54 In △ABC, AD bisects ∠A which meets BC at D. If BC = a, AC = b and AB = c, then DC = _______. 

A ab/(b+c)

B ac/(a+c)

C bc/(a+c)

D ac/(a+b)

Answer: A 

Q.55 If 9a2 + 4b2 + c2 + 21 = 4(3a + b − 2c), then the value of (9a + 4b – c) is: 

A 2 

B 16 

C 6 

D 12 

Answer: D 

Q.56 If sinθ = a/√(a2 +b2) ,0 < θ < 90, then the value of secθ + tanθ

A [√(a2 +b2)+a]/b

B [√(a2 +b2)+b]/2a

C [√(a2 +b2)+a]/2b

D [√(a2 +b2)+b]/a

Answer: A 

Q.57 The table shows the production of different types of cars (in thousands).

What is the ratio of the total production of type B cars in 2011 and type E cars in 2013 taken together to the total production of type C cars in 2014 and type D cars in 2012 taken together? 

A 8:9 

B 6:5 

C 5:6 

D 16:11 

Answer: B 

Q.58 A is 20% less than B while C is 20% more than D. If D is 25% less than A, then which of the following is true? 

A B = 0.675C 

B C = 0.72B 

C B = 0.72C 

D C = 0.675 B 

Answer: B 

Q.59 A person can row a distance of 4 km upstream in one hour 20 minutes and can row back to the starting point in just 24 minutes. How much time(in hours) will he take to row 13 km in still water? 

A 2 

B 3½

C 3 

D 2½

Answer: A 

Q.60 A person sold 25 articles for ₹2,500 and incurred a loss of 10%. How many articles should he sell for ₹2,400 to make a profit of 20%? 

A 15 

B 20 

C 16 

D 18 

Answer: D

Q.61 The table shows the production of different types of cars (in thousands). 

If the data related to the production of type C cars is represented by a pie-chart, then the central angle of the sector representing production of cars in 2012 will be: 

A 72 

B 73 ⅓ 

C 60 

D 80 

Answer: C 

Q.62 If (135√5 x3 − 2√2 y3 ) ÷ (3√5 x − 2 y) = Ax2 + By2 + 10 Cxy , then the value of (A + B – 9C) is: 

A 18 

B 12 

C 20 

D 10 

Answer: C 

Q.63 Ina circle of radius 10 cm and centre O, PQ and PR are two equal chords, each of length 12 cm. What is the length (in cm) of chord QR? 

A 18.6 

B 20.4 

C 18.4 

D 19.2 

Answer: D 

Q.64 If the 10-digit number 897359y7x2 is divisible by 72, then what is the value of(3x-y), for the possible greatest value of y ? 

A 3 

B 8 

C 7 

D 5 

Answer: C 

Q.65 The area of a field in the shape of a triangle with each side x metres is equal to the area of another triangular field having sides 50 m, 70 m and 80 m. The value of x is closest to: 

A 65.5 

B 63.2 

C 62.4 

D 61.8 

Answer: B 

Q.66 The value of 7½× ( 3⅕ ÷ 4½ of 5⅓) + [ 11 − (⅝ + 3 − 1¼)] ÷ 5¾ − 5 ÷ 5 × 5 of 5 ÷ 25 is: 

A ½

B ⅟₁₀

C ³/₁₀

D 1½

Answer: D 

Q.67 If x2 − 3x − 1 = 0, then the value of (x2 + 8x − 1)(x3 + x−1 ) −1 is: 

A ⅜

B 8 

C 1 

D 3 

Answer: C 

Q.68 The table shows the production of different types of cars (in thousands).

The average production of all types of cars in 2014 is approximately what percent less than the total production of type B cars in 2013 and type D cars in 2010 taken together? 

A 41.8% 

B 44.9% 

C 43.2% 

D 44.4% 

Answer: A 

Q.69 To do a certain work, A and B work on alternate days, with B beginning the work on the first day. A can finish the work alone in 48 days. If the work gets completed in days, then B alone can finish 4 times the same work in: 

A 24 days 

B 32 days 

C 27 days 

D 30 days 

Answer: C 

Q.70 If cos2 θ − 3cosθ + 2 = sin2 θ,0 < θ < 90, then the value of 2cosecθ + 4cotθ is: 

A ⁸/₃√3

B ⁴/₄√3

C 2√3

D 4√3

Answer: A 

Q.71 If (tanθ−sec θ+1)/(tanθ+secθ−1 ) secθ =⅟k, then k = 

A 1 + sinθ 

B 1 − cosθ

C 1 + cosθ

D 1 − sinθ

Answer: A 

Q.72 The table shows the production of different types of cars (in thousands). 

 

 

The total production of type A cars in 2011, and type C cars and type E cars in 2012 taken together is what percent of the total production of type B cars during 2010 to 2014? 

A 60.4% 

B 54.7% 

C 62.5% 

D 58.8% 

Answer: C 


Q.73 The total number of students in section A and B of a class is 110. The number of students in section A is 10 more than that of section B. The average score of the students in B,in a test, is 20% more than that of students in A. If the average score of all the students in the class is 72, then what is the average score of the students in A ? 

A 66 

B 68 

C 63 

D 70 

Answer: A 

Q.74 A shopkeeper sells an item for ₹492 after allowing 18% discount on its marked price. Had he NOT allowed any discount, he would have earned a profit of 20% on the cost price. What is the cost price of the item ? 

A ₹500 

B ₹640 

C ₹540 

D ₹600 

Answer: A 

Q.75 The prices of two articles are in the ratio 4 : 5. If the price of the first article is increased by x% and that of the other is decreased by 30%, then the new prices of A and B will be in the ratio 10 : 7. The value of x is:

A 24.5 

B 22.5 

C 25 

D 20 

Answer: C 

English 

Instructions 

In the following passage some words have been deleted. Fill in the blanks with the help of the alternatives given. Select the most appropriate option for each blank. 

Comprehension: During the reign of Aurangzeb, the last of the Mughals, there (1)______ revolts against the empire. (2)______ were the revolts of the Marathas, the Sikhs, the Jats and (3)______ others. After the death of Aurangzeb, the Mughal empire began to (4)______ fairly fast. The country soon (5)______ into smaller territories many of which became more or less independent. 

Q.76 Select the most appropriate option for blank No. 1 

A have been 

B was 

C are 

D were 

Answer: D 

Q.77 Select the most appropriate option for blank No. 2 

A These 

B It 

C That 

D This 

Answer: A 

Q.78 Select the most appropriate option for blank No. 3 

A more 

B many 

C each 

D every 

Answer: B 

Q.79 Select the most appropriate option for blank No. 4 

A disperse 

B disturb 

C disrupt 

D disintegrate 

Answer: D 

Q.80 Select the most appropriate option for blank No. 5 

A broke in 

B broke off 

C broke out 

D broke up 

Answer: D 

Instructions For the following questions answer them individually 

Q.81 In the sentence identify the segment which contains the grammatical error. The Japanese artist Yoh Nagao was busy splashing the wall from colours 

A splashing the wall 

B was busy 

C from colours 

D The Japanese artist 

Answer: C 

Q.82 Select the correct passive form of the given sentence. Please take the guest to his room on the 6th floor 

A The guest should be took to his room on the 6th floor 

B You are requested to take the guest to his room on the 6th floor 

C You must take the guest to his room on the 6th floor 

D Let the guest be taking to his room on the 6th floor 

Answer: B 

Q.83 Select the most appropriate option to substitute the underlined segment in the given sentence. If no substitution is required, select No improvement. His school was 7 km away from his house. I wondered how he covered so long distance daily on foot. 

A so long the distance 

B No improvement 

C such long distance 

D such a long distance 

Answer: D 

Q.84 Select the most appropriate word to fill in the blank. India is the second largest ______ of cotton in the world. 

A creator 

B producer 

C grower 

D manufacturer 

Answer: B 

Q.85 Select the word which means the same as the group of words given. a large, deep pot used both in the oven and as a serving vessel. 

A sauce-pan 

B casserole 

C skillet 

D cauldron 

Answer: B 

Q.86 Given below are four jumbled sentences. Select the option that gives their correct order. 

A. Since then Boeing has received over 5,000 orders for the aircraft and has delivered 371 to date. 

B. The Max first started flying commercially in 2017. 

C. Boeing had huge hopes for its 737 Max aircraft and viewed it as a key part of its future. 

D. However, the company’s market value has plummeted by nearly $26 billion since the crash in Ethiopia. 

A ABCD 

B CBAD 

C DCBA 

D BCDA 

Answer: C 

Q.87 Select the word which means the same as the group of words given. Flowers or leaves woven together in a circle for placing on a coffin or a grave 

A garland 

B wreath 

C bunch 

D bouquet 

Answer: B 

Q.88 In the sentence identify the segment which contains the grammatical error. The Doon Valley with all its lights look beautiful at night from the top of the mountain 

A look beautiful 

B from the top 

C at night 

D with all its lights 

Answer: A 

Q.89 Select the correct active form of the given sentence. The chairs were being arranged in the examination hall by the staff 

A The chairs were arranging the staff in the examination hall 

B The staff was arranging the chairs in the examination hall 

C The staff has been arranging the chairs in the examination hall 

D The staff has arranged the chairs in the examination hall 

Answer: B 

Q.90 Select the antonym of the given word. SOBER 

A nervous 

B agitated 

C serious 

D calm 

Answer: B 

Q.91 Select the wrongly spelt word. 

A negligible 

B nuisance 

C noticable 

D neighbouring 

Answer: C 

Q.92 Select the synonym of the given word. EXEMPT 

A hinder 

B exclude 

C reduce 

D prevent 

Answer: B 

Q.93 Select the most appropriate meaning of the given idiom. On tenterhooks 

A unhappy 

B angry 

C anxious 

D happy 

Answer: C 

Q.94 Select the most appropriate meaning of the given idiom. Lend an ear 

A to pay attention to 

B to not make trouble 

C to force someone to listen 

D to not tell someone something 

Answer: A

Q.95 Select the wrongly spelt word. 

A bridle 

B brisk 

C bristel 

D brittle 

Answer: C 

Q.96 Select the most appropriate option to substitute the underlined segment in the given sentence. If no substitution is required, select No improvement. We can’t live without water, will we? 

A won’t we 

B can we 

C do we 

D No improvement 

Answer: B 

Q.97 Select the synonym of the given word. RAMPANT 

A excessive 

B limited 

C rare 

D gentle 

Answer: A 

Q.98 Select the antonym of the given word. SWERVE 

A droop 

B lurch 

C deviate 

D straighten 

Answer: D 

Q.99 Select the most appropriate word to fill in the blank. The company management is of the opinion that not wearing a suit and tie at the workplace could create a bad ______ on clients. 

A response 

B notion 

C outlook 

D impression 

Answer: D 

Q.100 Given below are four jumbled sentences. Select the option that gives their correct order 

A. It has been a leading producer of both industrial and agricultural goods. 

B. The United States of America holds a dominant position in the world because of its high economic development. 

C. Because of its enormous output, the United States has a major share in world trade. 

D. It is also a leader in the development and application of innovative technology 

A BCAD 

B ABCD 

C BACD 

D DABC 

Answer: C 

SSC CGL Tier-I 10 June 2019 Shift-III Previous Year Paper

SSC CGL

(10 June 2019 Shift-III

Reasoning 

Instructions For the following questions answer them individually 

Q.1 Which two signs should be interchanged to make the given equation correct? 

16 + 3 − 5 × 2 ÷ 4 = 9 

A × and

B ÷ and +

C × and

D ÷ and ×

Answer: D 

 

Q.2 A square paper is folded and cut as shown below. How will it appear when unfolded ? 

Answer: A 

 

Q.3 Three of the following four letter-clusters are alike in a certain way and one is different. Pick the odd one out. 

A FHJ 

B SUW 

C KMO 

D PQR 

Answer: D 

 

Q.4 Select the figure in which the given figure is embedded. 

Answer: D 

 

Q.5 Three of the following four word pairs are alike in a certain way and one is different. Pick the odd word out. 

A Tamil : Kerala 

B Bori : Arunachal Pradesh 

C Konkani : Goa 

D Dogri : Jammuand Kashmir 

Answer: A 

 

Q.6 Find the next term in this series. CBA, FGE, ILI, LQM, ? 

A OVP 

B NVQ 

C OUP 

D OVQ 

Answer: D 

 

Q.7 Three different positions of a dice are shown below. Which number appears on the face opposite the number 4 ? 

A 6 

B 2 

C 5 

D 3 

Answer: C 

 

Q.8 Choose the Venn diagram from the given options which best represents the relationship amongst the following classes: Players, Singers, Students 

Answer: D 

 

Q.9 Select the option that is related to the third number in the same way as the second number is related to the first number. 17 : 102 :: 23 : ……… 

A 138 

B 256 

C 196 

D 152 

Answer: A 

 

Q.10 Select the correct mirror image of the given figure when the mirror is placed on the right of the figure. 

Answer: C 

 

Q.11 If SUN is coded as NUSand TOPis coded as POT, then which is the last letter in the code for the word FUR ?

A U 

B R 

C F 

D E 

Answer: C 

 

Q.12 Three of the following four letter-clusters are alike in a certain way and oneis different. Pick the odd one out. 

A KMT 

B FHO 

C QSZ 

D OQV 

Answer: D 

 

Q.13 Three of the following four word-pairs are alike in a certain way and one is different. Pick the odd one out. 

A Chisel : Sculptor 

B Anvil : Blacksmith 

C Paper : Book 

D Scalpel : Surgeon 

Answer: C 

 

Q.14 Select the term that will come next in the following series. 

5, 9, ?, 29, 45 

A 15 

B 25 

C 17 

D 19 

Answer: C 

 

Q.15 If DON is coded as 345 and ROAM is coded as 6412, then how will RANDOM be coded? 

A 615342 

B 651342 

C 615324 

D 613542 

Answer: A 

 

Q.16 A father was twelve times as old as his son twenty years ago. Now he is twice as old as his son. What are the present ages of the son and father? 

A 22 and 44 years 

B 27 and 54 years 

C 33 and 66 years 

D 15 and 30 years 

Answer: A 

 

Q.17 If + denotes — — denotes * * denotes / / denotes + then what will be the value of 

25 – 2 / 10 * 5 + 2 = ? 

A 10 

B 26 

C 58 

D 50 

Answer: D 

 

Q.18 Three of the following four number-pairs are alike in a certain way and one is different. Pick the odd one out. 

A 13 : 179 

B 8 : 69 

C 14 : 201 

D 12 : 149 

Answer: A 

 

Q.19 How many triangles are there in the following figure ? 

A 29 

B 18 

C 33 

D 31 

Answer: D

 

Q.20 ‘Fly’ is related to ‘Bird’ in the same way as ‘Gallop’ is related to ‘……….’ 

A Horse 

B Tiger 

C Lion 

D Elephant 

Answer: A 

 

Q.21 Select the figure that will come next in the following figure series. 

Answer: B 

 

Q.22 Bharat and Sapna are husband and wife. Rohit and Bharat are brothers. Suresh is the father of Rohit. Sapna’s son is Krish. How is Krish related to Suresh ? 

A Grandson 

B Son 

C Uncle 

D Father 

Answer: A 

 

Q.23 Find the missing number from the below options. 

A 140 

B 96 

C 112 

D 134 

Answer: C 

 

Q.24 Select the word pair in which the two words are related in the same way as the word pair given below. Feet : Socks 

A Head : Turban 

B Legs : Vest 

C Finger : Nail 

D Abdomen : Pants 

Answer: A 

 

Q.25 Select the option that is related to the third term in the same way as the second term is related to the first term. WOLF : FLOW :: DRAW : 

A RWAD 

B WRAD 

C WARD 

D DARW 

Answer: C 

 

General knowledge 

Instructions For the following questions answer them individually 

Q.26 As a reaction to Rowlatt Act, _________ was organised as National Humiliation Day. 

A 6th April 1919 

B 2nd February 1913 

C 14th June 1921 

D 8th May 1920 

Answer: A 

 

Q.27 Who was the first Indian to receive the Ramon Magsaysay Award for his contribution to community leadership? 

A Arvind Kejriwal 

B Baba Amte 

C Verghese Kurien 

D Acharya Vinoba Bhave 

Answer: D 

 

Q.28 ……….. was not only Sri Lanka’s first woman prime minister, but also the first woman prime minister in the world. 

A Upeksha Swarnamali 

B Sirimavo Bandaranaike 

C Rosy Senanayake 

D Chandrika Kumaratunga 

Answer: B 

 

Q.29 The Khilafat Movement of 1920 was organised as a protest against the injustice done to ______. 

A Turkey 

B Egypt 

C Iraq 

D Afghanistan 

Answer: A 

 

Q.30 In which year was the term ‘Gross Happiness Index’ coined by the fourth king of Bhutan, Jigme Singye Wangchuck? 

A 1964 

B 1989 

C 1972 

D 1982 

Answer: C 

 

Q.31 Mother Teresa, the founder of ‘Missionaries of Charity’, was born on ______. 

A  15th August 1910

B 26th May 1907 

C 13th January 1912 

D 2nd February 1913 

Answer: A 

 

Q.32 The Hindustan Socialist Republican Association (HSRA) was formed in the year _____ with an aim to overthrow the British. 

A 1930 

B 1928 

C 1921 

D 1922 

Answer: B

 

Q.33 The term ‘Putt’ is used in this sport ______. 

A Cricket 

B Football 

C Golf 

D Table Tennis 

Answer: C 

 

Q.34 How many provinces is the country of Nepal divided into? 

A 6 

B 4 

C 7 

D 5 

Answer: C 

 

Q.35 Which of the following is the busiest sea port in Pakistan? 

A Gwadar Port 

B Port of Qasim 

C Port of Karachi 

D Port of Keti 

Answer: C 

 

Q.36 How many ‘canine teeth’ does an adult human have? 

A 4 

B 2 

C 8 

D 3 

Answer: A 

 

Q.37 Who was the first female shooter from India to reach number 1 in world ranking by the International Shooting Sport Federation in 2014? 

A Shreyasi Singh 

B Anisha Sayyed 

C Anjali Bhagwat 

D Heena Sidhu 

Answer: D 

 

Q.38 Janku’ is a unique cultural custom followed in _______ as a celebration of life. 

A Pakistan 

B Nepal 

C Bhutan 

D Sri Lanka 

Answer: B 

 

Q.39 In 2018, Amnesty International stripped _________ of the Ambassador of Conscience Award given in 2009. 

A Benjamin Netanyahu 

B AI Gore 

C Aung San Suu Kyi 

D Henri Tiphagne 

Answer: C 

 

Q.40 What is the term used to describe the angular distance of a place north or south of Earth’s equator? 

A Pole 

B Longitude 

C Latitude 

D Hemisphere 

Answer: C 

 

Q.41 Which tribe of Pakistan performs a traditional dance form called ‘Waziri Dance’? 

A Sindhi 

B Brahuis 

C Baloch 

D Pashtun 

Answer: D

 

Q.42 The maximum strength of the Bangladeshi Parliament is ______. 

A 322 

B 363 

C 340 

D 350 

Answer: D 

 

Q.43 What is the definition for the term ‘hibernation’? 

A Building of habitat (nests) by birds to save themselves in rainy season 

B A habit of food conservation during summer season for winter among animals 

C A state of hyperactivity in spring time among birds 

D A state of reduced metabolic activity during winter season among some animals 

Answer: D 

 

Q.44 Which of the following plants is carnivorous? 

A Venus Flytrap 

B Hyacinth 

C Cypress Vine 

D Amaryllis 

Answer: A 

 

Q.45 Which of the following kings is regarded as the founder of the Kingdom of Nepal? 

A Tribhuvan Bir Bikram Shah 

B Mahendra Bir Bikram Shah Dev 

C Rana Bahadur Shah 

D Prithivi Narayan Shah 

Answer: D 

 

Q.46 A traditional ‘battery’ contains which of the following chemichals? 

A Ethanol 

B Ethylene glycol 

C Sodium bicarbonate 

D Sulphuric acid 

Answer: D 

 

Q.47 Which world leader is awarded the St Andrew the Apostle, the highest order of the Russian Federation in the year 2019? 

A Emmanuel Macron 

B Angela Markel 

C Narendra Modi 

D Donald Trump 

Answer: C 

 

Q.48 In economic terms what do we mean by ‘intermediate goods’? 

A Fixed assets used by manufacturers 

B Goods sold between industries for the resale or production of other goods 

C Price of goods without GST 

D Goods in transit before reaching the consumers 

Answer: B 

 

Q.49 Atal Tinkering Labs is an initiative by which of the following institutions? 

A National AYUSH Mission 

B NITI Aayog 

C Central Board of Secondary Education 

D Reserve Bank of India 

Answer: B 

 

Q.50 Which of the following is the third stage in the metamorphosis of a butterfly? 

A Pupa 

B Adult 

C Larva 

D Egg 

Answer: A 

 

Quantitative Aptitude 

Instructions For the following questions answer them individually 

Q.51 If x + 1/x= √5. Then x3 + 1/x3 is equal to: 

A 5√5

B 2√5

C 3√5

D 4√5

Answer: B 

 

Q.52 In a circle with centre O, AB is a diameter and CD is a chord such that ABCD is a trapezium. If ∠BAC = 15, then ∠CAD is equal to: 

A 30

B 75

C 60

D 45

Answer: C 

 

Q.53 If a : b = 4 : 5, then (2a + 3b) : (3a + 2b)is equal to: 

A 9 : 10 

B 22 : 33 

C 10 : 9 

D 23 : 22 

Answer: D 

 

Q.54 The price of sugar has increased by 18%. A person wants to increase the expenditure by 12% only. By what percent, correct to one decimal place, should he decrease his consumption? 

A 5.6% 

B 5.1% 

C 6% 

D 5.3% 

Answer: B 

 

Q.55 If a + b + c = 11 and ab + bc + ca = 38, then a3 + b3 + c3 − 3 3abc is equal to: 

A 66 

B 77

C 44 

D 55 

Answer: B 

 

Q.56 The average marks of 50 students in a class was found to be 64. If the marks of two students were incorrectly entered as 38 and 42 instead of 83 and 24, respectively, then what is the correct average? 

A 61.24 

B 64.54 

C 62.32 

D 61.86 

Answer: B 

 

Q.57 A train without stoppage travels with an average speed of 70km/h, and with stoppage,it travels with the average speed of 56km/h. How many minutes does the train stop on an average per hour? 

A 12 

B 14 

C 16 

D 15 

Answer: A 

 

Q.58 Two articles are sold for ₹10,384 each. On one,the seller gains 18% and onthe other, he loses 12%. What is his overall gain or loss? 

A ₹168 loss 

B ₹168 gain 

C ₹178 gain 

D ₹178 loss 

Answer: B 

 

Q.59 The table below shows the percentage of students and the ratio of boys and girls in different colleges. Total students = 1800 

What is the ratio of boys and girls in the colleges A and B taken together? 

A 45 : 71 

B 43 : 67 

C 67 : 104 

D 37 : 52 

Answer: C 

 

Q.60 The value of sin2 60 − cos2 45 + sec 60 + cos2 40 + cos2 50 is equal to: 

A 9/14

B 13/4

C 11/4

D 7/2 

Answer: B

 

Q.61 The difference between the compound interest and simple interest on ₹ x at 9% per annum for 2 years is ₹20.25. What is the value of x ? 

A 2,500 

B 2,400 

C 2,800 

D 2,200 

Answer: A 

 

Q.62 In △ABC, AD is median and is the point on AD such that AG : GD = 2: 1. Then ar(△ABG) : ar(△ABC) is equal to: 

A 1 : 6 

B 1 : 4 

C 1 : 3 

D 1 : 5 

Answer: C 

 

Q.63 when 2sin2 θ = 3cos θ, and 0 ≤ θ ≤ 90, then θ =?

A 60 

B 90 

C 30 

D 45

Answer: A 

 

Q.64 The table below shows the percentage of students and the ratio of boys and girls in different colleges. Total students = 1800 

In which college is the percentage difference between the number of boys and girls minimum? 

A C 

B A 

C D 

D E 

Answer: C 

 

Q.65 If the six digit number 6×2904 is divisible by 88, then the value ofx is: 

A 5 

B 8 

C 7 

D 6 

Answer: D 

 

Q.66 An article is sold for ₹642.60 after successive discounts of 15% and 10%. What is the marked price of the

article? 

A ₹820 

B ₹800 

C ₹880 

D ₹840 

Answer: D 

 

Q.67 If tanθ = ¾, then (4sinθ−cosθ)/(4sinθ+cosθ) is equal to: 

A ⅗ 

B ¼ 

C ½ 

D ⅖ 

Answer: C 

 

Q.68 The table below shows the percentage of students and the ratio of boys and girls in different colleges. Total students = 1800 

If 10%of the girls from college A are transferred to college E, then what is the increase in the percentage of girls in college E? 

A 4.6% 

B 4% 

C 4.2% 

D 4.4% 

Answer: B 

 

Q.69 If (3x − 1)3 + (4x − 3)3 + (2x + 1)3 = 3(3x − 1)(4x − 3)(2x + 1)  and x ≠ ⅓ then x = ?

A ½ 

B 1 

C 2 

D ¼ 

Answer: C 

 

Q.70 The radius of a sphere is increased by 140%. By what percent will its volume increase? 

A 174.4% 

B 1282.4% 

C 274.4% 

D 1382.4% 

Answer: B 

 

Q.71 △ABC ∼ △EDF and AB = 5 cm, BC = 8 cm and AC = 10 cm.If ar(△ABC) : ar(△DEF) = 9 : 4, then DFis equalto: 

A 16/3 cm 

B 20/3 cm 

C 32/9 cm 

D 10/3 cm 

Answer: A 

 

Q.72 The table below shows the percentage of students and the ratio of boys and girls in different colleges. Total students = 1800 

What is the percentage of girls in colleges D, E and F taken together, (nearest to one decimal place)? 

A 48.5% 

B 48.1% 

C 48.3% 

D 47.9% 

Answer: D 

 

Q.73 The value of 3.8 − (4.2 ÷ 0.7 × 3) + 5 × 2 ÷ 0.5 is: 

A 15.6 

B 5.8 

C 21.8 

D 18.4 

Answer: B 

 

Q.74 In a △ABC,the sides are AB = 16 cm, AC = 63 cm, BC = 65 cm. From A, a straight line AM is drawn up to the midpoint M of side BC. Then the length of AM is equal to: 

A 32.5 cm 

B 24.5 cm 

C 23.5 cm 

D 31.5 cm 

Answer: A 

 

Q.75 If x − 5√x − 1 = 0, then x2 + 1/x2 is equal to: 

A 625 

B 727 

C 729 

D 731 

Answer: B 

 

English 

Instructions For the following questions answer them individually 

Q.76 Given below are four jumbled sentences. Select the option that gives their correct order. 

A. Soon, flames ignited the wooden beams of the grand palace built by Xerxes. 

B. Men and women holding aloft flaming torches, raced up and down the palace terraces. 

C. When the fire died out, all that remained of the magnificent palace were the stone columns. 

D. Looters fought off the heat of the inferno to drag out gold and silver vessels. 

A ADBD 

B BADC 

C CDBA 

D BCAD 

Answer: B 

 

Instructions: In the following passage some words have been deleted. Fill in the blanks with the help of the alternatives given. Select the most appropriate option for each blank. 

Comprehension: Portia claims that even if she (1)______ to live as long as Sibylla, she would die as (2)______ as Diana because she can only be claimed (3)______ the manner specified by her father’s will. She (4)______ that nobody would be able to crack (5)______ casket test and so she was bound to remain unmarried. 

Q.77 Select the most appropriate option to fill in blank No.1. 

A were 

B is 

C was 

D has 

Answer: A 

 

Q.78 Select the most appropriate option to fill in blank No.2. 

A plain 

B pure 

C clear 

D real 

Answer: B 

 

Q.79 Select the most appropriate option to fill in blank No.3. 

A through 

B at 

C in 

D from 

Answer: C 

 

Q.80 Select the most appropriate option to fill in blank No.4. 

A has worry 

B worries 

C will worry 

D worry 

Answer: B 

 

Q.81 Select the most appropriate option to fill in blank No.5. 

A the 

B one 

C an 

D a 

Answer: A 

 

Instructions For the following questions answer them individually 

Q.82 Select the most appropriate option to substitute the underlined segment in the given sentence. If no substitution is required, select No improvement. It is convey to all the residents by now that they are required to apply for parking stickers. 

A It has been conveyed to all the residents 

B No improvement 

C It will be conveyed to all the residents 

D It is conveyed to all the residents 

Answer: A 

 

Q.83 Select the most appropriate synonym of the given word. REPUDIATE 

A renounce 

B sanction 

C enforce 

D regret 

Answer: A 

 

Q.84 Select the correctly spelt word. 

A accommodation 

B acomodation 

C accomodation 

D acommodation 

Answer: A 

 

Q.85 Select the most appropriate antonym of the given word. SEVERE 

A meticulous 

B morose 

C mild 

D mediocre 

Answer: C 

 

Q.86 Select the word which means the same as the group of words given. One who helps a person in need 

A samaritan 

B mercenary 

C collaborator 

D veteran 

Answer: A 

 

Q.87 Select the most appropriate option to fill in the blank. The owner of the house was ______ at the watchman for letting in a stranger. 

A impatient 

B indignant 

C indifferent 

D indulgent 

Answer: B 

 

Q.88 Select the most appropriate meaning of the underlined idiom in the given sentence. Extreme poverty made the poor woman wonder how long she could keep the wolf from the door. 

A avoid starvation 

B be safe in her house 

C keep wild animals away 

D live on charity 

Answer: A 

 

Q.89 In the sentence identify the segment which contains the grammatical error. Supriya asked Kiran that where had her mother gone when the results of the contest were being declared. 

A that where had her mother gone 

B when the results of the contest 

C were being declared 

D Supriya asked Kiran 

Answer: A 

 

Q.90 Select the most appropriate antonym of the given word. PRUDENT 

A practical 

B judicious 

C indiscreet 

D tactful 

Answer: C 

 

Q.91 Given below are four jumbled sentences. Select the option that gives their correct order. 

A. It was very unusual as boys were not supposed to be out of school at this late hour. 

B. He moved closer to the boy in anger so that he could recognise the miscreant and punish him. 

C. He felt angry as teachers ought to be about school rules being broken. 

D. Mr Oliver, the school teacher saw a lonely boy sitting on a rock, weeping soundlessly. 

A DBAC 

B ABCD 

C DACB 

D ACBD 

Answer: C 

 

Q.92 Select the most appropriate option to substitute the underlined segment in the given sentence. If no substitution is required, select No improvement. The world’s agricultural land are in pressure to raising more and more crops. 

A land is under pressure to raise 

B land is at pressure to raise 

C land is under pressure so raising 

D No improvement 

Answer: A 

 

Q.93 Select the most appropriate synonym of the given word. LAMENT 

A torment 

B afflict 

C distress 

D mourn 

Answer: D 

 

Q.94 Select the correct active form of the given sentence. You will either be taken prisoner or shot by the enemy. 

A The enemy will be either taking you prisoner or shooting you. 

B The enemy will either take you prisoner or shoot you. 

C The enemy would either take you prisoner or shoot. 

D The enemy either takes you prisoner or shoots you. 

Answer: B

 

Q.95 Select the most appropriate option to fill in the blank. I was frustrated at not being able to ______ of my old car. 

A depose 

B deal 

C devoid 

D dispose 

Answer: D 

 

Q.96 Select the word which means the same as the group of words given. Sole right to make and sell some invention 

A inheritance 

B heirloom 

C copyright 

D patent 

Answer: D 

 

Q.97 Select the correct passive form of the given sentence. Have they announced the world cup cricket team? 

A Has the world cup cricket team been announced? 

B Has the world cup cricket team being announced? 

C Have the world cup cricket team been announced? 

D Have the world cup cricket team announced? 

Answer: A 

 

Q.98 Select the most appropriate meaning of the underlined idiom in the given sentence. The ambitious project to impart free books to all students ended in smoke. 

A yielded no result 

B was delayed 

C exceeded the budget 

D was successfully completed 

Answer: A 

 

Q.99 In the sentence identify the segment which contains the grammatical error. On the way he was bited on his toe by a poisonous snake . 

A by a poisonous snake 

B he was bited 

C On the way 

D on his toe 

Answer: B 

 

Q.100 Select the correctly spelt word. 

A particuler 

B perticular 

C particular 

D perticuler 

Answer: C 

SSC CGL Tier-I 10 June 2019 Shift-II Previous Year Paper

SSC CGL 

(10 June 2019 Shift-II) 

Reasoning 

Instructions For the following questions answer them individually 

Q.1 Select the figure that will come next in the following figure series. 

Answer: A 

 

Q.2 Three of the following four words are alike in a certain way and one is different. Pick the odd word out. 

A Big 

B Expensive 

C Costly 

D Valuable 

Answer: A 

 

Q.3 How many triangles are there in the following figure ? 

A 34 

B 32 

C 36 

D 24 

Answer: A 

 

Q.4 Nisha and Deepak are a married couple and have a daughter named Tanya. Currently, Deepak is 5 years older than Nisha and Nishais thrice the age of Tanya. If Tanya is 10 years old, what washer father’s age at the time of his daughter’s birth? 

A 35 years 

B 25 years 

C 30 years 

D 20 years 

Answer: B 

 

Q.5 Find the missing number from the below options. 

A 96 

B 76 

C 58 

D 29 

Answer: C 

 

Q.6 Which two signs should be interchanged in the following equation to make it correct ?

8 × 2 + 5 − 16 ÷ 4 = 14 

A × and

B × and +

C ÷ and × 

D ÷ and +

Answer: B 

 

Q.7 Select the word pair in which the two words are related in the same way as the two words in the following word-pair. Season : Winter 

A Week : Calendar 

B Summer : Autumn 

C Year : Century 

D Month : April 

Answer: D 


Q.8 Three of the following four number-pairs are alike in a certain way and one is different. Pick the odd one out. 

A 56 : 49 

B 64 : 54 

C 104 : 91 

D 32 : 28 

Answer: B 

 

Q.9 Find the missing number from the below options. 

A 175 

B 164 

C 185 

D 154 

Answer: A 

 

Q.10 In a code language, COMPUTER is written as IVGFKNLX. How will TELEPHONE be written in that language? 

A VMNSKVOVG 

B GVOVKSLMV 

C VMLSKUOVG 

D VMLSKVOVG 

Answer: D 

 

Q.11 Select the term that will come next in the following series. 

3, 4, 5, 6, 9, 10, 11, 12, 15, ? 

A 16 

B 17 

C 14 

D 18 

Answer: A 

 

Q.12 Select the Venn diagram that best represents the relationship among the given set of classes. Students, Teachers, School 

Answer: D 

 

Q.13 A square paper is folded and cut as shown below. How will it appear when unfolded ? 

Answer: D 

 

Q.14 Which letter will replace the question mark (?) in the following series? 

A, B, E, J, Q, ? 

A A 

B Y 

C X 

D Z 

Answer: D 

 

Q.15 Select the option that is related to the third number in the same way as the second number is related to the first number. 12 : 68 :: 21 : ? 

A 49 

B 117 

C 119 

D 79 

Answer: C 

 

Q.16 Saksham introduced Nidhi to his friend, “She is the daughter of the only son of my father’s wife.” How is Saksham related to Nidhi? 

A Son 

B Cousin 

C Father 

D Brother 

Answer: C 

 

Q.17 Select the option that will come next in the following series. 

BOP, DPN, FQL, HRJ, ? 

A JSH 

B JSI 

C ISH 

D ITI 

Answer: A 

 

Q.18 Three different positions of a dice are shown below. Which number appears on the face opposite number 3 ? 

A 5 

B 6 

C 2 

D 4 

Answer: A 

 

Q.19 Three of the following four letter-clusters are alike in a certain way and one is different. Pick the odd one out. 

A POWZ 

B KNQT 

C FILO 

D QTWZ 

Answer: A

 

Q.20 Two statements are given followed by two conclusions numbered I and II. Assuming the statements to be true, even if they seem to beat variance with commonly known facts, decide which of the conclusions logically follow(s) from the statements. 

Statements: 

Some teachers are philosophers. 

Some philosophers are writers. 

Conclusions: 

I. Some writers are teachers. 

II. No writer is a teacher. 

A Only conclusion II follows 

B Either conclusion I or II follows 

C Neither conclusion I nor II follows 

D Only conclusion I follows 

Answer: B 

 

Q.21 If EAGER is coded as 51759 then how will CADET be coded? 

A 31457 

B 34157 

C 31547 

D 31450 

Answer: A 

 

Q.22 Arrange the following words in a logical and meaningful order. 1) Egypt 2) Africa 3) Great Pyramid 4) World 5) Giza 

A 3, 5, 2, 1, 4 

B 3, 5, 1, 2, 4 

C 5, 3, 1, 2, 4 

D 5, 3, 1, 4, 2 

Answer: B 

 

Q.23 Select the correct mirror image of the given figure when the mirror is placed to the right of the figure. 

Answer: A 

 

Q.24 Select the figure in which the given figure is embedded. 

Answer: A 

 

Q.25 ‘China’ is related to ‘Yuan’ in the same way as ‘Japan’ is related to ‘…….’. 

A Rand 

B Sushi 

C Lira 

D Yen 

Answer: D 

 

General knowledge 

Instructions For the following questions answer them individually 

Q.26 Who among the following was the first Indian origin recipient of the prestigious Pulitzer Prize? 

A Geeta Anand 

B Gobind Behari Lal 

C Siddhartha Mukherjee 

D Jhumpa Lahiri 

Answer: B 

 

Q.27 Of which Indian state is ‘Gamocha’ a cultural symbol? 

A Kerala 

B Assam 

C Rajasthan 

D Haryana 

Answer: B 

 

Q.28 How many members are present in the Sri Lankan parliament? 

A 210 

B 215 

C 232 

D 225 

Answer: D 


Q.29 What is the popular name of ‘Ascorbic Acid’? 

A Vitamin A 

B Vitamin B12 

C Vitamin K 

D Vitamin C 

Answer: D 

 

Q.30 Viyahula Giddha’ is a popular folk dance performed during marriages in the Indian state of _______ . 

A Odisha 

B Gujarat 

C Punjab 

D Jharkhand 

Answer: C 

 

Q.31 Who became the first Indian player to represent India at the Wimbledon Tennis Championship of Independent India? 

A Ramesh Krishnan 

B Vijay Amritraj 

C Leander Paes 

D Ramanathan Krishnan 

Answer: D 

 

Q.32 The explosion of crackers is an example of ___________. 

A Combustion 

B Precipitation 

C Decomposition 

D Evaporation 

Answer: A 

 

Q.33 World Global Recycling Day was observed on ______ in 2019. 

A 2nd January 

B 10th February 

C 18th March 

D 16th February 

Answer: C 

 

Q.34 Who among the following was the first president of Pakistan? 

A Zulfikar Ali Bhutto 

B Yahya Khan 

C Iskander Mirza 

D Ayub Khan 

Answer: C 

 

Q.35 An economic condition when there is one buyer and many sellers is called ______. 

A Oligopoly 

B Monopoly 

C Perfect Competition 

D Monopsony 

Answer: D 

 

Q.36 Which of the following is a vestigial organ? 

A Heart 

B Kidney 

C Lungs 

D Appendix 

Answer: D 

 

Q.37 Which of the following countries has the largest parliament in the world? 

A UK 

B India 

C China 

D Japan 

Answer: C 

 

Q.38 Which one of the following is the brightest star in the Orion Constellation? 

A Betelgeuse 

B Alnilam 

C Rigel 

D Eta Orionis 

Answer: C 

 

Q.39 What is the dominant chemical present in vinegar? 

A Malic acid 

B Formic acid 

C Sulphuric acid 

D Ethanoic acid 

Answer: D 

 

Q.40 Which of the following Indian islands lies in the Bay of Bengal? 

A Andaman and Nicobar 

B Lakshadweep 

C Diu 

D Daman 

Answer: A 

 

Q.41 Mahatma Gandhi formed the Natal Indian Congress in the year ______. 

A 1874 

B 1894 

C 1854 

D 1863 

Answer: B 

 

Q.42 What is the geographic name given to a deep and narrow valley consisting of steep sides created by weathering and erosion by rivers, wind, rain and tectonic activity? 

A Buttes 

B Ridge 

C Canyon 

D Basin 

Answer: C 

 

Q.43 The First Anglo-Burmese War ended with the signing of the ______. 

A Treaty of Yandabo 

B Treaty of Titalia 

C Treaty of Purandar 

D Treaty of Salbai 

Answer: A 

 

Q.44 Who among the following is the author of the book ‘A Passage to England’? 

A Nirad C Chaudhuri 

B V S Naipaul 

C Salman Rushdie 

D Khushwant Singh 

Answer: A 

 

Q.45 Who authored ‘A Century is not Enough’in 2018? 

A Sourav Ganguly 

B Sachin Tendulkar 

C MS Dhoni 

D Virat Kohli 

Answer: A 

 

Q.46 Who among the following became the first woman to win the Abel Prize 2019? 

A Sophie Germain 

B Maryam Mirzakhani 

C Karen Uhlenbeck 

D Julia Robinson 

Answer: C 

 

Q.47 In which year did Independent India win its first Olympic Gold in the game of hockey? 

A 1952 

B 1960 

C 1948 

D 1956 

Answer: C 

 

Q.48 Silent Valley National Park is located in the Indian state of ________. 

A Madhya Pradesh 

B West Bengal 

C Kerala 

D Uttarakhand 

Answer: C 

 

Q.49 The Bangladesh Liberation War ended on _____. 

A 14th November 1972 

B 16thDecember 1971 

C 17th October 1971 

D 2nd October 1974 

Answer: B 

 

Q.50 What is the name given to the graph that shows all the combinations of two commodities that a consumer can afford at given market prices and within the particular income level in economic terms? 

A Demand Curve 

B Isocost Line 

C Supply Curve 

D Budget Line 

Answer: D 

 

Quantitative Aptitude 

Instructions For the following questions answer them individually 

Q.51 An article is sold for ₹528 after successive discounts of 20% and 12%. What is the marked price of the article? 

A ₹760 

B ₹740 

C ₹750 

D ₹780 

Answer: C 

 

Q.52 The table below shows the number of students enrolled in five colleges over the five years (2010 to 2014). 

What is the average number of students studying in college D over the given years? 

A 450 

B 420 

C 430 

D 440 

Answer: A 

 

Q.53 A train without stoppage travels with an average speed of 50 km/h, and with stoppage, it travels with an average speed of 40 km/h. For how many minutes does the train stop on an average per hour? 

A 12 

B 13 

C 14 

D 15 

Answer: A 

 

Q.54 If x + 1/x= 3, then x3 + 1/x3 is equal to: 

A 27 

B 36 

C 24 

D 18 

Answer: D 

 

Q.55 A is 40% more efficient than B and C is 20% less efficient than B. Working together, they can finish a work is 5 days. In how many days will A alone complete 70% of that work? 

A 9 

B 7 

C 10 

D 8 

Answer: D 

 

Q.56 If tanθ = ⅔, then (3sinθ−4cosθ)/(3sinθ+4cosθ) is equal to: 

A -⅓

B ⅔

C -⅔

D ⅓

Answer: A 

 

Q.57 If sec4θ = cosec(θ + 20 ), then θ is equal to: 

A 22 

B 18 

C 14 

D 20 

Answer: C 

 

Q.58 In a circle with centre O, AB is the diameter and CD is a chord such that ABCD is a trapezium. If ∠BAC = 40 , then ∠CAD is equal to: 

A 15 

B 20 

C 50 

D 10 

Answer: D 

 

Q.59 The value of sin2 38 + sin2 52 + sin2 30 − tan2 45 is equal to: 

A ⅓

B ¼

C ¾

D ½

Answer: B 

 

Q.60 △ABC~△RQP and AB = 4 cm, BC = 6 cm and AC = 5 cm. If ar(△ABC) : ar(△PQR) = 9 : 4, then PQ is equal to: 

A ²⁰/₉cm 

B ⁸/₃cm 

C 4 cm 

D ¹⁰/₃cm 

Answer: C

 

Q.61 The table below shows the number of students enrolled in five colleges over the five years (2010 to 2014). 

In the year 2014, what percent of students were enrolled in college C (correct to one decimal place)? 

A 16.9% 

B 17.3% 

C 16.7% 

D 17.1% 

Answer: A 

 

Q.62 The radius of a sphere is reduced by 40%. By what percent will its volume decrease? 

A 60% 

B 64% 

C 72.5% 

D 78.4% 

Answer: D 

 

Q.63 If √x 1/√x= 4, then x2 + 1/x2 is equal to: 

A 192 

B 326 

C 322 

D 256 

Answer: C 

 

Q.64 The table below shows the number of students enrolled in five colleges over the five years (2010 to 2014). 

What is the ratio of the total students enrolled in colleges A and B in the year 2012 to the total students enrolled in colleges D and E in the year 2013? 

A 62 : 85 

B 62 : 88 

C 63 : 86 

D 58 : 63 

Answer: A 

 

Q.65 The price of sugar is increased by 20%. A person wants to increase his expenditure by 8% only. By what percent should he decrease his consumption? 

A 10% 

B 11% 

C 9% 

D 12% 

Answer: A 

 

Q.66 The value of 4.5 − (3.2 ÷ 0.8 × 5) + 3 × 4 ÷ 6 is: 

A -13.5 

B 4.2 

C -8.5 

D 5.7 

Answer: A 

 

Q.67 From a point P outside a circle, PAB is a secant and PTis a tangent to the circle, where, A, B and T are points on the circle. If PT = 5 cm, PA = 4 cm and AB = x cm,then x is equal to: 

A 2.25 cm 

B 2.75 cm 

C 2.45 cm 

D 1.75 cm 

Answer: A 

 

Q.68 The average marks of 40 students was found to be 68. If the marks of two students were incorrectly entered as 48 and 64 instead of 84 and 46 respectively, then what is the correct average? 

A 68.25 

B 68.15 

C 68.45 

D 68.35 

Answer: C 

 

Q.69 If a + b + c = 13 and ab + bc + ca = 54, then a3 + b3 + c3 − 3abc is equal to: 

A 793 

B 273 

C 91 

D 182 

Answer: C 

 

Q.70 If a : b = 3 : 2, then (5a + 2b) : (3a + 4b) is equal to: 

A 16:15 

B 8:7 

C 19:17 

D 17:14 

Answer: C 

 

Q.71 Two articles are sold for ₹9,720 each. On one, the seller gains 8% and on theother, he loses 10%. What is his overall gain or loss? 

A ₹380 gain 

B ₹380 loss 

C ₹360 loss 

D ₹360 gain 

Answer: C 

 

Q.72 The difference between the compound interest and simple interest on ₹ x at 8% per annum for 2 years is ₹19.20. What is the value of x? 

A 2,500 

B 3,200 

C 2,800 

D 3,000 

Answer: D 

 

Q.73 The table below shows the number of students enrolled in five colleges over the five years (2010 to 2014). 

The number of students studying in college E in the year 2013 is approximately what percent of the number of students studying in colleges B, C and D taken together in the year 2013 (nearest to one decimal place)? 

A 38.2% 

B 38.6% 

C 39.1% 

D 39.4% 

Answer: C 


Q.74 In △ABC, AD is the median and G is a point on AD such that AG : GD = 2 : 1. Then ar( △BDG): ar( △ABC) is equal to: 

A 1 : 4 

B 1 : 9 

C 1 : 6 

D 1 : 3 

Answer: C 

 

Q.75 If the six digit number 15x1y2 is divisible by 44, then (x + y) is equal to: 

A 8 

B 7 

C 6 

D 9 

Answer: B 

 

English 

Instructions For the following questions answer them individually 

Q.76 Select the most appropriate antonym of the given word. ACCEPT 

A reject 

B acquire 

C obtain 

D release 

Answer: A 

 

Q.77 Select the most appropriate option to fill in the blank. Technology can be ______ and people seem to be more attached to screens than ever before. 

A addictive 

B infatuated 

C incorrigible 

D contagious 

Answer: A 

 

Q.78 Select the correctly spelt word. 

A challeng 

B challange 

C chalenge 

D challenge 

Answer: D 

 

Q.79 Select the correct active form of the given sentence. Let the bell be rung every forty minutes. 

A The bell should be rung every forty minutes 

B Ring the bell every forty minutes. 

C The bell ought to be rung every forty minutes 

D Let the bell keep ringing every forty minutes. 

Answer: B 

 

Q.80 Select the word which means the same as the group of words given. Something which is fit to be eaten 

A unpalatable 

B delicious 

C edible 

D tasty 

Answer: C 

 

Q.81 Select the most appropriate option to substitute the underlined segment in the given sentence. If no substitution is required, select No improvement. Biologists believe that increased human activity means an accelerated rate of change in habitat for all creatures of the world. 

A No improvement 

B human activity meant a acelerated rate in change 

C human activities mean an acelerated rate from change 

D human activity is meaning an acelerated rate of change 

Answer: A 

 

Q.82 Given below are four jumbled sentences. Select the option that gives their correct order. 

A. Here, the picture perfect French Quarter and heritage laden Tamil streets blend into one another. 

B. The conflict of coexistence is evident in many metropolitan cities. 

C. The old and new, like two different worlds, surviving side by side yet never crossing that chasm. 

D. There is a place though- Puducherry, where contrasts are celebrated like nowhere else. 

A BACD 

B BCDA 

C DCBA 

D CDAB 

Answer: B 

 

Q.83 Select the word which means the same as the group of words given. Something which cannot be understood 

A illegible 

B incomprehensible 

C inexplicable 

D infallible 

Answer: B 

 

Q.84 Select the correct passive form of the given sentence. He is clicking good pictures with his new camera. 

A Good pictures were clicked with his new camera. 

B Good pictures have been clicked with his new camera. 

C Good pictures are being clicked with his new camera. 

D Good pictures are clicked with his new camera. 

Answer: C 

 

Q.85 In the sentence identify the segment which contains the grammatical error. No sooner did he see the tiger when he ran as fast as he could. 

A no sooner did 

B he see the tiger 

C when he ran 

D as fast as he could 

Answer: C 

 

Q.86 Select the most appropriate synonym of the given word. PENSIVE 

A spontaneous 

B spiteful 

C reflective 

D tragic 

Answer: C 

 

Q.87 Select the most appropriate meaning of the underlined idiom in the given sentence. Coming from an affluent family, she found herself a square peg in a round hole when she married a poor farmer and moved to a small village. 

A in a favourable situation 

B unhappy and regretful 

C a misfit in the environment 

D in a financial crisis 

Answer: C 

 

Q.88 Select the most appropriate synonym of the given word. VINDICTIVE 

A watchful 

B revengeful 

C forceful 

D helpful 

Answer: B 

 

Q.89 Select the most appropriate meaning of the underlined idiom in the given sentence. When his envious competitor extended a hand of friendship, he smelt a rat. 

A detected something wrong 

B felt very pleased 

C became complaisant 

D behaved arrogantly 

Answer: A 

 

Q.90 In the sentence identify the segment which contains the grammatical error. The match is about to begin since the captain as well as the team are on the field. 

A are on the field 

B The match is about to begin 

C since the captain 

D as well as the team 

Answer: A 

 

Q.91 Given below are four jumbled sentences. Select the option that gives their correct order. 

A. European cloth manufacturers therefore had to depend on a plant called woad to make dyes. 

B. However, cloth dyers preferred indigo which produced a rich blue colour. 

C. Being a plant of the temperate zones, woad was easily available in Europe. 

D. Indian indigo was very expensive and only small quantities reached the European markets. 

A CBDA 

B DCAB 

C ACBD 

D DACB 

Answer: D 

 

Q.92 Select the most appropriate antonym of the given word. AFFINITY 

A aversion 

B attraction 

C empathy 

D preference 

Answer: A 

 

Q.93 Select the most appropriate option to substitute the underlined segment in the given sentence. If no substitution is required, select No improvement. A reason why there are so much misconception on dyslexia could be the sheer invisibility of the disorder. 

A there are so much misconceptions against dyslexia 

B there are so many misconceptions about dyslexia 

C there are so many misconception around dyslexia 

D No improvement 

Answer: B 

 

Instructions: In the following passage some words have been deleted. Fill in the blanks with the help of the alternatives given. Select the most appropriate option for each blank. 

Comprehension: Telling someone something that you admire about them can be called a random act (1)______ kindness. It takes almost no (2)______, yet it pays enormous dividends. Many people (3)______ their entire lives wishing that others would (4)______ them. Expectations are more from family and friends, (5)______, even compliments from strangers feel good if they are genuine. 

Q.94 Select the most appropriate option to fill in blank No.1. 

A with 

B of 

C from 

D in 

Answer: B

 

Q.95 Select the most appropriate option to fill in blank No.2. 

A venture 

B work 

C effort 

D attempt 

Answer: C 

 

Q.96 Select the most appropriate option to fill in blank No.3. 

A spent 

B had spent 

C have been spending 

D spend 

Answer: D 

 

Q.97 Select the most appropriate option to fill in blank No.4. 

A confess 

B acknowledge 

C concede 

D admit 

Answer: B 

 

Q.98 Select the most appropriate option to fill in blank No.5. 

A so 

B however 

C and 

D unless 

Answer: B 

 

Instructions For the following questions answer them individually 

Q.99 Select the most appropriate option to fill in the blank. People of the older generation ______ over the good old school days. 

A remember 

B remind 

C retribute 

D reminisce 

Answer: D 

 

Q.100 Select the correctly spelt word. 

A concentious 

B conscentious 

C conscientious 

D consenteous 

Answer: C 

SSC CGL Tier-I 10 June 2019 Shift-I Previous Year Paper

SSC CGL

(10 June 2019 Shift-I) 

Reasoning 

Instructions For the following questions answer them individually 

Q.1 Select the set in which the numbers are related in the same way as are the numbers of the following set.

(17, 12, 7) 

A (42, 34, 27) 

B (23, 32, 39) 

C (15, 19, 23) 

D (39, 28, 19) 

Answer: C 

 

Q.2 Select the set in which the numbers are related in the same way as are the numbers of the following set. (3, 24, 7) 

A (8, 80, 12) 

B (10, 71, 13) 

C (9, 63, 14) 

D (6, 75, 11) 

Answer: A 

 

Q.3 Arrange the following words in a logical and meaningful order. 

1. Child 2. Marriage 3. Infant 4. Employment 5. Education 

A 4, 1, 3, 5, 2 

B 3, 1, 5, 4, 2 

C 3, 5, 2, 4, 1 

D 5, 2, 4, 1, 3 

Answer: B 

 

Q.4 ‘Geology’ is related to ‘Rocks’ in the same way as ‘Cytology’is related to ‘ …… ’. 

A Organism

B Cells 

C Plants 

D Life 

Answer: B 

 

Q.5 Select the word-pair in which the two words are related in the same way as are the two words in the following word pair. Barter : Commodities 

A Treat : Disease 

B Debate : Discussion 

C Movement : Traffic 

D Correspond : Letters 

Answer: D 

 

Q.6 Select the figure in which the given figure is embedded. 

Answer: A 

 

Q.7 The average of 9 numbers is 40. If the average of the first five numbers is 38 and that of the last five is 50, then what is the fifth number? 

A 90 

B 84 

C 78 

D 80 

Answer: D 

 

Q.8 Three of the following four numbers are alike in a certain way and one is different. Pick the number that is different from the rest. 

A 953 

B 523 

C 312 

D 734 

Answer: A 

 

Q.9 Select the option that is related to the third letter-cluster in the same way as the second letter-cluster is related to the first letter-cluster. DGIL : WTRO :: FHKM : ? 

A USPN 

B TRON 

C QPLI 

D WTSP 

Answer: A 

 

Q.10 A square paper is folded and cut as shown below. How will it appear when unfolded ? 

Answer: A 

 

Q.11 If HEAD is coded as 37 and BANK is coded as 57, then how will KITE be coded as? 

A 87 

B 91 

C 69 

D 76 Answer: B 

 

Q.12 Which two signs should be interchanged in the following equation to make it correct? 

24 − 12 ÷ 4 + 8 × 2 = 11 

A + and × 

B − and ÷ 

C + and − 

D + and ÷ 

Answer: C 

 

Q.13 Which number will replace the question mark (?) in the following series? 16, 24, 36, ?, 81 

A 54 

B 52 

C 61 

D 58 

Answer: A 

 

Q.14 Select the number-pair in which the two numbers are related in the same way as are the two numbers of the following number-pair. 24 : 56 

A 18:48 

B 12:36 

C 9:21 

D 15:40 

Answer: C 

 

Q.15 Ina code language, PASTEUR is coded as TPRUASE. How would SEVENTY be coded in that language? 

A ENVETYS 

B NSYTEVN 

C ESYETNV 

D ESYTEVN 

Answer: D 

 

Q.16 Ina family of seven persons, B is the brother of A and the son of C.D is the son-in-lawof C, who has two grand children, E and F. A is the mother of F, who is the niece of G. E is the son of G. If C has two children, how is E related D? 

A Brother-in-law 

B Son 

C Cousin 

D Nephew 

Answer: D 

 

Q.17 Select the figure that will come next in the following figure series. 

Answer: D 

 

Q.18 Select the mirror image of the given figure when the mirror is placed to the right of the figure. 

Answer: C 

 

Q.19 How many triangles are there in the following figure ? 

A 14 

B 24 

C 22 

D 18 

Answer: C

 

Q.20 Three of the following four words are alike in a certain way and one is different. Pick the odd word out. 

A Steal 

B Hinder 

C Impede 

D Arrest 

Answer: A 

 

Q.21 Select the combination of letters that when sequentially placed in the gaps of the given letter series will complete the series. ca_bab_ad_ab_ad_ab 

A bcdcb 

B cdbcd 

C dcbcb 

D dcabc 

Answer: C 

 

Q.22 Two different positions of the same dice are shown. Which number will beat the top if 5 is at the bottom ? 

A 6 

B 4 

C 2 

D 3 

Answer: B 

 

Q.23 Two statements are given, followed by three conclusions numbered I, II and III. Assuming the statements to be true even if they seem to be at variance with commonly known facts, decide which of the conclusions logically follow(s) from the statements. Statements: 

Some carpenters are technicians. 

All technicians are mechanics. 

Conclusions: 

I. Some carpenters are mechanics. 

II. Some mechanics are technicians. 

III. All carpenters are mechanics. 

A None of the conclusions follows 

B Only conclusion I follows 

C Only conclusions I and II follow 

D Only conclusions II and III follow 

Answer: C 

 

Q.24 Three of the following four letter-clusters are alike in a certain way and one is different. Pick the odd one out. 

A MLKP 

B TSRN 

C XVWZ 

D DCBZ 

Answer: C 

 

Q.25 Select the Venn diagram that best illustrates the relationship between the following classes. Indians, Females, Voters 

Answer: A 

 

General knowledge 

Instructions For the following questions answer them individually 

Q.26 The Central Board of Secondary Education (CBSE) announced the introduction of skill subjects in the school curriculum for the academic session 2019-20. Which of the following is NOT one of them? 

A Spiritual Enhancement 

B Artificial Intelligence (AI) 

C Yoga 

D Early Childhood Care Education (ECCE) 

Answer: A 

 

Q.27 One of the prominent Buddhist structures in India, Stupa at Sarnath was constructed by the great Mauryan king, Ashoka. 

A Dhauli 

B Dhamekh 

C Bharhut 

D Lalitgiri 

Answer: B 

 

Q.28 Which of the following elements is an actinide? 

A Ytterbium 

B Lutetium 

C Curium 

D Erbium 

Answer: C 

 

Q.29 Law of octaves says that if the chemical elements are arranged according to increasing atomic weight, those with similar physical and chemical properties occur after each interval of elements. 

A 10 

B 5 

C 7 

D 9 

Answer: C 

 

Q.30 A situation where the expenditure of the government exceeds its revenueis called ………… 

A Default Financing 

B Deficit Revenue 

C Budget Deficit 

D Default Revenue 

Answer: C 

 

Q.31 The Global Business Summit, launched on 15 January 2015, is a flagship initiative of the ……… Group. 

A Economist 

B Bhaskar 

C Times 

D Dutch 

Answer: C 

 

Q.32 Special Olympics, held every year, is a global movement of people creating a new world of inclusion, where every single person is accepted and welcomed regardless of their ability or disability. 

A five 

B two 

C three 

D four 

Answer: B

 

Q.33 The power to summon the Houses of the Parliament is vested with the ……….. 

A Prime Minister 

B Speaker 

C Vice President 

D President 

Answer: D 

 

Q.34 Superstars, a new platform to analyse the game of cricket, comprises three metrics. Which of the following is NOT one of them? 

A Score Index 

B Luck Index 

C Smart Stats 

D Forecaster 

Answer: A 

 

Q.35 In February 2019, Flight Lieutenant …….. created history by becoming the first Indian woman flight engineer. 

A Mohana Singh 

B Hina Jaiswal 

C Bhawana Seth 

D Anjali Gupta 

Answer: B 

 

Q.36 Safdarjung’s tomb, set in the middle of a garden, was built by Nawab Shuja-ud-Daulah in ………. 

A Uttar Pradesh 

B Uttarakhand 

C Delhi 

D Bihar 

Answer: C 

 

Q.37 ……….. married Mehr-un-Nisa whom he gave the title of ‘Nur Jahan’ (light of the world). 

A Shah Jahan 

B Aurangzeb 

C Akbar 

D Jahangir 

Answer: D 

 

Q.38 Which of the following elements is a lanthanide? 

A Actinium 

B Francium 

C Cerium 

D Polonium 

Answer: C 

 

Q.39 ……….. pass connects Uttarakhand and Tibet andis situated in the north of Gangotri. 

A Zoji La 

B BaraLacha La 

C Shipki La 

D Muling La 

Answer: D 

 

Q.40 The Parliament of India consists of ………. 

A President, House of the People and Council of States 

B House of the People 

C House of the People and Council of States 

D Council of States 

Answer: A 

 

Q.41 In his periodic table, Mendeleev could NOT assign correct position to ……….. 

A Oxygen 

B Carbon 

C Nitrogen 

D Hydrogen 

Answer: D

 

Q.42 A nautical mile is equal to ………… 

A 2000 metres 

B 1852 metres 

C 1672 metres 

D 2450 metres 

Answer: B 

 

Q.43 ……….. , activist-journalist from Karnataka, who was shot dead in September 2017, was posthumously conferred the Anna Politkovskaya Award 2017. 

A Gauri Lankesh 

B Naveen Gupta 

C Sudip Dutta Bhaumik 

D Sayed Mehdi 

Answer: A 

 

Q.44 ………… Strait separates the islands of Java (east) and Sumatra. 

A Sunda 

B Yucatan 

C Malacca 

D Cook 

Answer: A 

 

Q.45 ……….. died in 1605, nearly 50 years after his ascension to the throne. He was buried outside of Agra at Sikandra. 

A Akbar 

B Aurangzeb 

C Shah Jahan 

D Jahangir 

Answer: A 

 

Q.46 Swaroop Rawal, an Indian teacher at Lavad Primary School in ……… was in the running for the prestigious Global Teacher Prize which honours the world’s best teacher. 

A Gujarat 

B Maharashtra 

C Kerala 

D Tamil Nadu 

Answer: A 

 

Q.47 ……… has been called the “architect king” as during his reign, the world witnessed a unique development of arts and culture of the Mughal Empire. 

A Shah Jahan 

B Jahangir 

C Aurangzeb 

D Akbar 

Answer: A 

 

Q.48 It was under the reign of ………. that the Mughal Empire reached its peak in matter of area. 

A Jahangir 

B Aurangzeb 

C Shah Jahan 

D Akbar 

Answer: B 

 

Q.49 Who among the following gave the ‘Law of Octaves’? 

A Lavoisier 

B Mendeleev 

C Newlands 

D Dobereiner 

Answer: C 

 

Q.50 Indian origin campaigner ……… was named UK’s Most Influential Black Person of the Year on 25 October 2017. 

A Priti Patel 

B Gina Miller 

C Jonathan Marland 

D David Olusoga 

Answer: B 

 

Quantitative Aptitude 

Instructions For the following questions answer them individually 

Q.51 The table shows the production of different types of cars (in thousands). 

The average production of type A cars during the five years is what percent of the total production of type C cars during the five years? 

A 22.4 

B 20.6 

C 21.8 

D 18.7 

Answer: A 

 

Q.52 If 4x2 − 6x + 1 = 0, then the value of 8x3 + (8x3 )−1 is: 

A 36 

B 13 

C 18 

D 11 

Answer: C 

 

Q.53 The table shows the production of different types of cars (in thousands). 

 

What is the ratio of the total production of type C and D cars in 2012 to the total production of type A cars in 2014 and type E cars in 2015? 

A 5 : 6 

B 9 : 11 

C 3 : 4 

D 11 : 12 

Answer: C 

 

Q.54 The average of twelve numbers is 55.5. The average of the first four numbers is 53.4 and that of the next four numbers is 54.6. The 10th number is greater than the 9th number by 3 but lesser than the 11th and 12th numbers by 2 and 3, respectively. What is the average of the 10th and the 12th numbers ? 

A 59.5 

B 58 

C 57.5 

D 56 

Answer: A 

 

Q.55 The table shows the production of different types of cars (in thousands). 

The total production of type B cars in 2015 and type D cars 2016 is what percent less than the total production of type E cars in five years? 

A 50⅓ 

B 52 ⅔

C 46⅔

D 53⅓ 

Answer: D 

 

Q.56 The speed of train A is 25 km/h more than the speed of train B. A takes 4 hours less time to travel a distance of 300 km than what train B takes to travel 250 km. what is the speed (in km/h) of A? 

A 60 

B 50 

C 65 

D 55 

Answer: B 

 

Q.57 A circle touches the side PQ of a △APQ at the point R and sides AP and AQ produced at the points B and C, respectively. If the perimeter of △APQ = 30 cm, then the length of AB is: 

A 20 cm 

B 10 cm 

C 12 cm 

D 15 cm 

Answer: D 

 

Q.58 The income of A is 40% more than that of B. If A got a 25% rise in his income and B got a 40% rise in his income, then the percentage increase in the combined incomes of A and B is: 

A 31.25 

B 34.5 

C 28.25 

D 24.5 

Answer: A 

 

Q.59 Asum of ₹12,000 amounts to ₹20,736 in 3 years at a certain rate percent per annum, interest compounded annually. What will amount of the same sumto in 2 years at the same rate on compound interest? 

A ₹15,640 

B ₹17,820 

C ₹17,280 

D ₹14,520 

Answer: C 

 

Q.60 The marked price of an article is ₹550. A shopkeeper allows a discount of 20% and still gets a profit of 10%. If he sells it for ₹470, his profit percent will be: 

A 16.8 

B 18 

C 17.5 

D 16 

Answer: C

 

Q.61 The value of  ¾×2⅔÷⁵/₉ of 1⅕+2⅔×3⅚÷²/₇ of 2⅓ is: 

A 1⅚

B 1⅔

C ½

D ⅚

Answer: C 

 

Q.62 In a △ABC,right angled at B, AB = 7 cm and (AC – BC) = 1 cm. The value of (secC + cotA) is : 

A 1 5 6

B 43

C 3 4

D 1 

Answer: B 

 

Q.63 The curved surface area and the volume of a cylinder are 264cm2 and 924cm3 , respectively. What is the ratio of its radius to height ?(Take π = ²²/₇) 

A 4:3 

B 5:4 

C 7:6 

D 3:2 

Answer: C 

 

Q.64 The ratio of the efficiencies of A, B and C, to do a certain work is 7 : 3 : 5. Working together, they can complete the work in 21 days. A and C worked together for 15 days. The remaining work will be completed by B alone in: 

A 54 days 

B 45 days 

C 60 days 

D 63 days 

Answer: B 

 

Q.65 The table shows the production of different types of cars (in thousands). 

If the data related to the production of type B cars is represented by a pie chart, then the central angle of the sector representing the production of cars in 2016 is: 

A 56∘ 

B 81∘ 

C 75∘ 

D 73∘ 

Answer: B 

 

Q.66 A bought an article for ₹5,400 and sold it at a loss of 30%. With this amount, he bought another article and sold it at a gain of 60%.What was his overall percentage gain or percentage loss? 

A Gain, 1.2% 

B Gain, 12% 

C Loss, 12% 

D Loss, 1.2% 

Answer: B 

 

Q.67 In △PQR, QT ⊥ PR and S is a point on QR such that PSQ = p∘ . If ∠TQR = 46 and ∠SPR = 32, then the value of p is: 

A 78 

B 82 

C 76 

D 72 

Answer: C 

 

Q.68 If 3sinθ = 2 cos2 θ,0 < θ < 90 , then the value of (tan2 θ + sec2 θ − cosec2 θ) is: 

A -2 

B −37 

C 7 3

D 2 

Answer: B 

 

Q.69 If (tanθ+sinθ)/(tanθ-sinθ)= (k+1)/(k-1), then k = ? 

A cosecθ 

B secθ 

C cosθ 

D sinθ 

Answer: B 

 

Q.70 If x + y + z = 0, then the value of (x2 + y2 + z2 ) ÷ (z2 − xy) is: 

A 1 

B 2 

C -2 

D -1 

Answer: B 

 

Q.71 What is the ratio of the mean proportional between 4.8 and 10.8 and the third proportional to 0.4 and 2.4? 

A 2 : 1 

B 3 : 2 

C 1 : 2 

D 2 : 3 

Answer: C 

 

Q.72 In △ ADC, E and B are the points on the sides AD and AC respectively such that ∠ ABE = ∠ ADC. If AE = 6 cm,BC = 2 cm, BE = 3 cm and CD = 5 cm, then (AB + DE)is equal to: 

A 14 cm 

B 16 cm 

C 493cm 

D 463cm 

Answer: D 

 

Q.73 Let O be the centre of a circle and AC be its diameter. BD is a chord intersecting AC at E. Point A is joined to B and D. If ∠ BOC = 50∘ and ∠ AOD = 110∘ , then ∠ BEC = ? 

A 80∘ 

B 70∘ 

C 55∘ 

D 90∘ 

Answer: A 

 

Q.74 If a2 + b2 + c2 + 27 = 6(a + b + c) , then what is the value of 3 √(a3 + b3 − c3

A 3 

B 1 

C 9 

D 6 

Answer: A 

 

Q.75 If an 11-digit number 5y5884805x6, x ≠ y, is divisible by 72, then the value of √(xy) is: 

A √7 

B 3 

C 7 

D 2√7 

Answer: C 

 

English 

Instructions For the following questions answer them individually 

Q.76 Select the correct passive form of the given sentence. The enemy will have seized the fort before night fall. 

A The fort would have been seized by the enemy before nightfall. 

B The enemy will be seized by the fort before nightfall 

C The fort will have been seized by the enemy before nightfall. 

D The enemy will seize the fort before nightfall. 

Answer: C 

 

Q.77 Given below are four jumbled sentences. Select the option that gives their correct order. 

A. For a split second, as the stranger stepped into the light, his furry head was clearly visible. B. Atfirst, all seemed still outside the house,except for the sound of the rain. 

C. Recognizing him, Joe stood petrified staring at the man wearing a long coat,wet from rain. 

D. But then, footsteps approached the house and a stranger emerged out of the darkness. 

A CABD 

B BCDA 

C DACB 

D BDAC 

Answer: D 

 

Q.78 Select the most appropriate antonym of the given word. NOTORIOUS 

A famous 

B vicious 

C infamous 

D disgraceful 

Answer: A 

 

Q.79 Select the correctly spelt word. 

A maintenance 

B mentainance 

C maintenence 

D maintainence 

Answer: A 

 

Q.80 Select the most appropriate option to substitute the underlined segment in the given sentence. If no substitution is required, select No improvement. Though many European traders visit Puducherry in the sixteenth century onwards, what remains todayis a relic of its French past. 

A No improvement 

B traders visited Puducherry in a sixteenth century onward 

C traders visited Puducherry from the sixteenth century onwards 

D trader visit Puducherry from the sixteenth century onwards 

Answer: C 

 

Q.81 Select the most appropriate antonym of the given word. SACRED 

A perpetual 

B pious 

C profane 

D profound 

Answer: C 

 

Instructions: Select the most appropriate meaning of the underlined idiom in the given sentence. 

Q.82 It is unfortunate that most people in the country are still living from hand to mouth. 

A consuming daily whatlittle is earned 

B saving for the future generations 

C doing manual labour 

D earning wealth by selfish means 

Answer: A 

 

Q.83 The loyal watchman was cut to the quick when he was accused of theft. 

A imprisoned 

B severely punished 

C hurt intensely 

D dismissed 

Answer: C 

 

Instructions For the following questions answer them individually 

Q.84 In the sentence identify the segment which contains the grammatical error. Most disputes can be solved amicably unless one are not rigid. 

A Most disputes 

B can be solved amicably 

C unless one are 

D not rigid 

Answer: C 

 

Q.85 Select the correct active form of the given sentence. She was seen sitting in the last row. 

A We see her sitting in the last row. 

B We saw her sitting in the last row. 

C We had seen her sitting in the last row. 

D We have seen her sitting in the last row. 

Answer: B 

 

Q.86 Select the word which means the same as the group of words given. One who is indifferent to art and culture 

A cynic 

B scientist 

C philanderer 

D philistine 

Answer: D 

 

Q.87 Select the most appropriate option to substitute the underlined segment in the given sentence. If no substitution is required, select No improvement. The India consensus study highlight a benefits by adding domestic violence prevention approaches to current government policy. 

A highlight the benefits from added 

B highlight a benefit in addition 

C No improvement 

D highlights the benefits of adding 

Answer: D 

 

Q.88 Select the word which means the same as he group of words given. One who embraces voluntary death for the sake of one’s country 

A fanatic 

B martyr 

C patriot 

D diplomat 

Answer: B 

 

Q.89 Given below are four jumbled sentences. Select the option that gives their correct order.

A. The students often go about their business, singing along in the corridors.

B. But for the students of St. Ambrose High School, it no longer dictates their day.

C. A bell can sound gloomy or cheerful depending on when it is rung.

D. It has been scrapped in favour of music which now breaks up the time table of the school . 

A CADB 

B BDAC 

C CBDA 

D ACBD 

Answer: C 

 

Instructions: Select the most appropriate synonym of the given word. 

Q.90 PRECARIOUS 

A abundant 

B dangerous 

C valuable 

D premature 

Answer: B 

 

Q.91 STERILE 

A pure 

B productive 

C barren 

D sordid 

Answer: C 

 

Instructions For the following questions answer them individually 

Q.92 In the sentence identify the segment which contains the grammatical error. In the northern suburbs of Bengaluru, home to the bulk of the information technology industry, the water crisis is even worst. 

A home to the bulk of 

B is even worst 

C the water crisis 

D In the northern suburbs 

Answer: B 

 

Instructions In the following passage some words have been deleted. Fill in the blanks with the help of the alternatives given. Select the most appropriate option for each blank. 

Comprehension: Perhaps India should listen to her (1) ………… sages who taught the importance (2)…………. balance. This means we should be (3) ……….. not to take anything too far (4) …………. be too radical. The unfortunate results of (5) ………. too far are obvious in the environmental degradation. 

Q.93 Select the most appropriate option to fill in blank No.1. 

A extinct 

B ancient 

C initial 

D antique 

Answer: B 

 

Q.94 Select the most appropriate option to fill in blank No.2. 

A for 

B in 

C of 

D by 

Answer: C

 

Q.95 Select the most appropriate option to fill in blank No.3. 

A careless 

B careful 

C carefree 

D caretaker 

Answer: B 

 

Q.96 Select the most appropriate option to fill in blank No.4. 

A but 

B otherwise 

C so 

D or 

Answer: D 

 

Q.97 Select the most appropriate option to fill in blank No.5. 

A venturing 

B pioneering 

C climbing 

D risking 

Answer: A 

 

Instructions For the following questions answer them individually 

Q.98 Select the correctly spelt word. 

A adolescence 

B adolesense 

C adolescense 

D adolescance 

Answer: A 

 

Instructions Select the most appropriate option to fill in the blank. 

Q.99 She performs different roles very convincingly as she is a very ………. actor. 

A virtuous 

B voracious 

C versatile 

D verbose 

Answer: C 

 

Q.100 Charles Dickens was different in his style of writing from most of his ………. in the Victorian era. 

A contemporaries 

B cronies 

C critics 

D confidants 

Answer: A 

SSC CGL Tier-I 11 June 2019 Shift-III Previous Year Paper

SSC CGL

(11 June 2019 Shift-III) 

Reasoning 

Instructions For the following questions answer them individually 

Q.1 Select the figure that will come next in the following figure series. 

Answer: D 

 

Q.2 Select the figure in which the given figure is embedded. (Rotation is not allowed) 

Answer: C 

 

Q.3 Three of the following four letter-clusters are alike in a certain way and one is different. Pick the odd one out. 

A PUYBD 

B EJNQS 

C TYBEG 

D JOSVX 

Answer: C 

 

Q.4 In a code language, SELECTION is written as ESELDITNO. How will NOSTALGIA be written in the same language? 

A ONTSBGLAI 

B ONTSZGLAI 

C NOTSBLGAI 

D ONSTBGLAI 

Answer: A 

 

Q.5 The sum of the current ages of Natasha and Krishna is 50 years. 10 years ago, Krishna was twice as old as Natasha. What is Krishna’s current age? 

A 15 years 

B 20 years 

C 10 years 

D 30 years 

Answer: D 

 

Q.6 Three different positions of the same dice are shown below. Which symbol is on the face opposite the face showing ’*’ ? 

A + 

B % 

C # 

D & 

Answer: D 

 

Q.7 Select the letter cluster that will come next in the following series. 

ADH, XAE, UXB, RUY, ? 

A PRV 

B ORU 

C OSV 

D ORV 

Answer: D 

 

Q.8 Arrange the following words in a logical and meaningful order. 

1. Kidney 2. Heart 3. Brain 4. Thyroid gland 5. Liver 

A 3, 4, 2, 5, 1 

B 3, 2, 4, 5, 1 

C 3, 4, 2, 1, 5 

D 3, 2, 4, 1, 5 

Answer: A 

 

Q.9 Select the correct mirror image of the given figure when a vertical mirror is placed on the right of the figure. 

Answer: D 

 

Q.10 Two statements are given, followed by three conclusions numbered I, II and II. Assuming the statements to be true, even if they seem to be at variance with commonly known facts, decide which of the conclusions logically follow(s) from the statements. Statements: 

All squirrels are lizards. 

All lizards are goose. 

Conclusions: 

I. Some squirrels are not goose. 

II. Some lizards are squirrels. 

III. Some goose are squirrels. 

A Conclusions I and II follow 

B Conclusions I, II and III follow 

C Conclusions I and III follow 

D Conclusions II and ITI follow 

Answer: D 

 

Q.11 Given below is a transparent sheet of paper with a pattern on the left and right side. How will the pattern appear when the transparent sheet is folded on the dotted line ? 

Answer: B 

 

Q.12 Select the number that will replace ? in the following series. 

120, 128, 144, 168, ?, 240 

A 216 

B 202 

C 208 

D 200 

Answer: D 

 

Q.13 A + B means ‘A is the mother of B’ 

A – B means ‘A is the father of B’ 

A ×B means ‘A is the sister of B’ 

A ÷ B means ‘A is the daughter of B’

If P + R × T – Q ÷ S + U, then how is S related to T ? 

A Daughter 

B Sister 

C Mother 

D Wife 

Answer: D 

 

Q.14 Which two signs should be interchanged to make the given equation correct? 

27 ÷ 3 − 18 + 3 × 2 = 18 

A ÷ and × 

B + and ÷ 

C + and − 

D × and

Answer: B 

 

Q.15 Select the word-pair in which the two words are related in the same way as the two words in the following word-pair. Walnut : Fig 

A Almond : Dry fruits 

B Turmeric : Spices 

C Asafoetida : Cashew nuts 

D Raisin: Apricot 

Answer: D 

 

Q.16 How many triangles are there in the figure given below ? 

A 32 

B 36 

C 24 

D 30 

Answer: D 

 

Q.17 In a code language, if PICTURE is coded as 16932021185 then how will FUNCTION be coded in the same language? 

A 6211432091514 

B 6211432091413 

C 6211332091513 

D 6211432081514 

Answer: A 

 

Q.18 Find the missing number from the below options. 

A 9 

B 51 

C 12 

D 3 

Answer: D 

 

Q.19 Select the option that is related to the third number in the same way as the second number is related to the first number. 16 : 240 :: 6 : ? 

A 25 

B 30 

C 35 

D 40 

Answer: B

 

Q.20 Three of the following four words are alike in a certain way and one is different. Pick the odd word out. 

A Honesty 

B Integrity 

C Truthfulness 

D Hypocrisy 

Answer: D 

 

Q.21 Select the term that will replace the question mark (?) in the following series. 

A, G, L, ?, S, U, V 

A O 

B P 

C Q 

D N 

Answer: B 

 

Q.22 In the Venn diagram given below,the circle represents ‘Ladies’, the triangle represents ‘Constables’ and the square represents ‘Married persons’. The numbers given in the diagram represents number of persons of that particular category. 

How many lady constables are NOT married? 

A 14 

B 12 

C 8 

D 15 

Answer: B 

 

Q.23 Three of the following four number-pairs are alike in a certain way and one is different. Pick the odd one out. 

A 4 : 16 

B 14 : 196 

C 12 : 244 

D 2 : 4 

Answer: C 

 

Q.24 ‘Insulin’ is related to ‘Hormone’ in the same way as ‘Pepsin’ is related to ‘ ………. ’ 

A Digestion 

B Vitamins 

C Stomach 

D Enzymes 

Answer: D 

 

Q.25 Find the missing number from the below options. 

A 19 

B 20 

C 21 

D 18 

Answer: B 

 

General knowledge 

Instructions For the following questions answer them individually 

Q.26 The ‘Tanakh’ is the sacred text of which religion/sect? 

A Taoism 

B Confucianism 

C Zen Buddhism 

D Judaism 

Answer: D 

 

Q.27 In which year was the ‘Battle of Goa’ fought? 

A 1510 

B 1502 

C 1524 

D 1514 

Answer: A 

 

Q.28 The phrase ‘Survival of the fittest’ as a way of describing the mechanism of natural selection was coined by ______. 

A Marie Curie 

B Charles Babbage 

C Herbert Spencer 

D Louis Pasteur 

Answer: C 

 

Q.29 In which province of China is the Huangguoshu National Park located which houses the world’s largest waterfall cluster. 

A Yunnan 

B Jiangsu 

C Guizhou 

D Shandon 

Answer: C 

 

Q.30 Bhavai’ is a traditional dance form of ______. 

A Punjab 

B Maharashtra 

C Bihar 

D Gujarat 

Answer: D 

 

Q.31 DNA is stored majorly in ______ of the cell. 

A plasma membrane 

B nucleus 

C cytoplasm 

D golgi body 

Answer: B 

 

Q.32 The ‘Friendship Highway’ is a road that connects China to ______. 

A Pakistan 

B Myanmar 

C Nepal 

D India 

Answer: C

 

Q.33 The former princely state of Tripura in the north-eastern part of India was ruled by _________ Dynasty. 

A Ahom 

B Nagvanshi 

C Manikya 

D Haihaya 

Answer: C 

 

Q.34 Which of the following was a port city of the Indus Valley Civilization? 

A Kalibangan 

B Rakhigarhi 

C Dholavira 

D Lothal 

Answer: D 

 

Q.35 Cash Reserve Ratio (CRR) is calculated as a percentage of each bank’s _____. 

A savings of customers 

B credit growth 

C rate of inflation 

D net demand and time liabilities 

Answer: D 

 

Q.36 The Indus Water Treaty was signed between India and ______. 

A China 

B Bangladesh 

C Pakistan 

D Afghanistan 

Answer: C 

 

Q.37 In which part of the female flower does fertilisation take place? 

A Ovary 

B Filament 

C Style 

D Stigma 

Answer: A 

 

Q.38 Who among the following is the only Indian to have won the Amateur World title in both, snooker and billiards? 

A Michael Ferreira 

B Pankaj Advani 

C Subhash Agarwal 

D Geet Sethi 

Answer: B 

 

Q.39 World Water Day’ is annually celebrated on ______. 

A 2nd June 

B 13th August 

C 15th January 

D 22nd March 

Answer: D 

 

Q.40 When was the Constitution of Pakistan enforced? 

A 1965 

B 1973 

C 1947 

D 1952 

Answer: B 

 

Q.41 Harsh Vardhan, the union minister for Environment, Forest and Climate Change launched the India Cooling Action Plan (ICAP). On which day was ICAP released? 

A 26th January 

B 8th March 

C 15th February 

D 27th February 

Answer: B 

 

Q.42 Night Blindness is caused by the deficiency of Vitamin _______. 

A A 

B C 

C K 

D B12

Answer: A 

 

Q.43 About 70% of the sun is made up of ______. 

A Helium 

B Hydrogen 

C Oxygen 

D Carbon 

Answer: B 

 

Q.44 Which Indian author wrote the book ‘The English Teacher’? 

A Ruskin Bond 

B R K Narayan 

C Khushwant Singh 

D Vikram Seth 

Answer: B 

 

Q.45 Which one of the following is the highest altitude lake of Pakistan? 

A Satpara Lake 

B Paristan Lake 

C Attabad Lake 

D Rush Lake 

Answer: B 

 

Q.46 The interest rate charged by banks on short-term loans to their largest, most secure and most creditworthy customers is called ______. 

A Prime Lending Rate 

B Variable Rate 

C Amortised Rate 

D Discount Rate 

Answer: A 

 

Q.47 The Constitution of India was amended for the first time in which year? 

A 1954 

B 1961 

C 1951 

D 1960 

Answer: C 

 

Q.48 _____ assumed the title of ‘Gangaikondachola’ or the conqueror of the river Ganga. 

A Rajaraja Chola I 

B Vijjayalay Chola 

C Rajendra Chola I 

D Rajadhiraja Chola 

Answer: C 

 

Q.49 Which of the following Indian authors is one of the four screenplay writers of the movie ‘Kai Po Che’? 

A Chetan Bhagat 

B Ravinder Singh 

C Amish Tripathi 

D Durjoy Datta 

Answer: A 

 

Q.50 ____________ became the most successful doubles player in the history of Davis Cup as on 2018. 

A Mark Woodforde 

B Henri Leconte 

C Leander Paes 

D Leon Smith 

Answer: C 

 

Quantitative Aptitude 

Instructions For the following questions answer them individually 

Q.51 The value of sin2 48 + sin2 42 − sec2 30 + tan2 60is equal to: 

A 2 

B 7/2

C 8/3

D 5/3

Answer: C 

 

Q.52 For what value of x is the seven digit number 46393×8 divisible by 11 ? 

A 3 

B 2 

C 7 

D 5 

Answer: A 

 

Q.53 If a : b = 5 : 8 and c : b = 4 : 3, then a : b : c is equal to: 

A 15 : 24 : 32 

B 15 : 24 : 28 

C 5 : 6 : 8 

D 5 : 8 : 6 

Answer: A 

 

Q.54 In △ABC,P is a point on BC such that BP : PC = 1 : 2 and Q is the midpoint of BP. Then, ar( △ABQ): ar( △ABC) is equal to: 

A 1 : 6 

B 1 : 4 

C 1 : 5 

D 1 : 3 

Answer: A 

 

Q.55 The difference between the compound interest and simple interest on ₹x at 12% per annum for 2 years is ₹18. What is the value of x? 

A 1,250 

B 1,280 

C 1,340 

D 1,300 

Answer: A 

 

Q.56 If a + b = 8 and ab = 32/3, then (a3 + b3 )is equal to: 

A 128 

B 256 

C 384 

D 320 

Answer: B 

 

Q.57 If a + b + c = 4 and ab + bc + ca = 2, then a3 + b3 + c3 − 3abc is equal to: 

A 32 

B 48 

C 40 

D 36 

Answer: C 

 

Q.58 A is 50% more efficient than B and C is 40% less efficient than B. Working together, they can complete a task in 10 days. In how many days will A alone complete 150% of that task? 

A 31 

B 33 

C 28 

D 35 

Answer: A 

 

Q.59 If √x + 1/√x= √7 , then x3 + 1/x3 is equal to: 

A 110 

B 130 

C 120 

D 140 

Answer: A 

 

Q.60 If secθ = 13/5, then tanθ − sinθ + cosθ is equal to: 

A 23/13

B 118/65 

C 121/65 

D 124/65 

Answer: C

 

Q.61 This table shows the number of students studying in various streams in different colleges. 

What is the ratio of the number of students studying science in colleges A and B together to the number of students studying commerce in colleges D and E together? 

A 13 : 8 

B 23 : 15 

C 21 : 17 

D 26 : 17 

Answer: D 

 

Q.62 If sin5θ = cos(50 − 3θ), then θ is equal to: 

A 25 

B 15 

C 20 

D 30 

Answer: C 

 

Q.63 Ina circle with centre O, AB is the diameter and CD is a chord such that ABCD is a trapezium. If ∠BAC = 24 , then ∠CAD is equal to: 

A 48 

B 36 

C 42 

D 24 

Answer: C 

 

Q.64 The price of sugar is increased by 21%. A person wants to increase his expenditure by 12% only. By what percent, correct to one decimal place, should he reduce his consumption? 

A 7.6% 

B 7.2% 

C 7.8% 

D 7.4% 

Answer: D 

 

Q.65 Two chords AB and CD of a circle intersect at a point P inside the circle. If AB = 7 cm, PC = 2 cm and AP = 4 cm,then CD is equal to: 

A 4 cm 

B 6 cm 

C 8 cm 

D 5 cm 

Answer: C 

 

Q.66 train without stoppage travels with an average speed of 80 km/h and with stoppage,it travels with an average speed of 72 km/h. For how many minutes does the train stop on an average per hour? 

A 6 

B 8 

C 9 

D 7 

Answer: A 

 

Q.67 This table shows the number of students studying in various streams in different colleges. 

The number of students in the science stream of college C is approximately what percentage of students studying in that college? 

A 43% 

B 42% 

C 44% 

D 45% 

Answer: D 

 

Q.68 The radii of the two circular faces of the frustum of a cone of height 14 cm are 5 cm and 2 cm. What is its volume in cm3 (π = 22/7) 

A 540 

B 572 

C 520 

D 560 

Answer: B 

 

Q.69 △ABC∼△QPR and AB = 8 cm, BC = 12 cm and AC = 6 cm. If ar( △ABC) : ar( △PQR) = 16 : 25, then RQ is equal to: 

A 10 cm 

B 15 cm 

C 7.5 cm 

D 12.5 cm 

Answer: C 

 

Q.70 This table shows the number of students studying in various streams in different colleges. 

What is the average of the number of students in the arts stream in all the colleges taken together? 

A 440 

B 460 

C 450 

D 470 

Answer: A 

 

Q.71 The value of: 5.8 + (7.4 ÷ 3.7 × 5) − 6 × 2 ÷ 2.5 

A 12 

B 11 

C 9 

D 10 

Answer: B 

 

Q.72 An article is sold for ₹535.50 after two successive discounts of 25% and 15%. What is the marked price of the article? 

A ₹820 

B ₹800 

C ₹840 

D ₹830 

Answer: C 

 

Q.73 In a class of 50 students, 46% are girls and the remaining are boys. The average of the boys’ marks is 58 and that of the girls is 62. What are the average marks of the whole class? 

A 60.38 

B 60.65 

C 59.84 

D 60.12 

Answer: C 

 

Q.74 This table shows the number of students studying in various streams in different colleges. 

If the data about students of the commerce stream in all colleges is represented by a pie-chart, what is the central angle of the sector representing college D, to the nearest degree? 

A 85 

B 88 

C 80 

D 82 

Answer: A 

 

Q.75 Two articles are sold for ₹2508 each. On one, there is a gain of 14% and on the other, there is a loss of 12%. What is the overall gain or loss percent to the nearest one decimal place? 

A 0.5% gain 

B 0.7% loss 

C 0.7% gain 

D 0.5% loss 

Answer: B 

 

English 

Instructions For the following questions answer them individually 

Q.76 Select the most appropriate option to substitute the underlined segment in the given sentence. If no substitution is required, select No improvement. She was so tired from working any longer. 

A so tired to not work 

B so tiring to work 

C No improvement 

D too tired to work 

Answer: D 

 

Q.77 In the sentence identify the segment which contains the grammatical error. He is smarter than his brother though he does not earn much money. 

A though he does not earn 

B much money 

C He is more smarter 

D than his brother 

Answer: C 

 

Instructions: Select the most appropriate option to fill in the blank. 

Q.78 The accident victim ______ to his injuries before he could be taken to the hospital. 

A surrendered 

B succumbed 

C subscribed 

D submitted 

Answer: B 

 

Q.79 We should never ______ with the rules of driving. 

A temper 

B trifle 

C reckon 

D tamper 

Answer: D 

 

Instructions For the following questions answer them individually 

Q.80 Select the most appropriate synonym of the given word. EXALT 

A praise 

B condemn 

C challenge 

D extract 

Answer: A 

 

Q.81 Select the correctly spelt word. 

A manipulate 

B manipulete 

C menipulate 

D manepulate 

Answer: A 

 

Q.82 Select the correct passive form of the given sentence It is time to serve dinner. 

A It is time for dinner to be served. 

B It is time that dinner is serve. 

C It is dinner serving time. 

D It is time so dinner should be served. 

Answer: A 

 

Q.83 Select the most appropriate antonym of the given word. VIGILANT 

A visible 

B careless 

C careful 

D elusive 

Answer: B 

 

Q.84 Select the correct active form of the given sentence. Superstitions are still believed in by people. 

A People still believed in superstitions. 

B People are still believing in superstitions. 

C People have still believed in superstitions. 

D People still believe in superstitions. 

Answer: D 

 

Q.85 Select the word which means the same as the group of words given. The act of looking back on past time 

A introspection 

B inspection 

C circumspection 

D retrospection 

Answer: D 

 

Q.86 Select the correctly spelt word. 

A arguement 

B argeument 

C arguemant 

D argument 

Answer: D 

 

Q.87 Select the most appropriate antonym of the given word. EXTRAVAGANT 

A thrifty 

B deficit 

C pauper 

D generous 

Answer: A 

 

Q.88 In the sentence identify the segment which contains the grammatical error. The child along with his parents were waiting for the programme to begin. 

A The child 

B along with his parents 

C were waiting for the 

D programme to begin 

Answer: C 

 

Q.89 Given below are four jumbled sentences. Select the option that gives their correct order. 

A. The first of these states that the only way to have a friend is to be one. 

B. Like all relationships, it is also governed by some well-defined principles. 

C. Friendship is one of the most coveted relationships on earth. 

D. Thus, one must be reliable as a friend to expect reliability in others. 

A BDAC 

B CBAD 

C ACBD 

D CDBA 

Answer: B 

 

Instructions: In the following passage some words have been deleted. Fill in the blanks with the help of the alternatives given. Select the most appropriate option for each blank. 

Comprehension: The Bhopal gas tragedy has been described as the world’s (1)______ industrial disaster. Forty two tonnes of methyl isocyanate (2)______ from the steel containers (3)______ the Union Carbide factory and released a cloud of (4)______ gas.It left a legacy of instant and (5)______ death. 

Q.90 Select the most appropriate option to fill in blank No.1 

A worse 

B bad 

C worst 

D best 

Answer: C 

 

Q.91 Select the most appropriate option to fill in blank No.2 

A leaked 

B flowed 

C released 

D emitted 

Answer: A 

 

Q.92 Select the most appropriate option to fill in blank No.3 

A beside 

B along 

C at 

D across 

Answer: C 

 

Q.93 Select the most appropriate option to fill in blank No.4 

A merciless 

B deathly 

C abominable 

D incurable 

Answer: B 

 

Q.94 Select the most appropriate option to fill in blank No.5 

A deferred 

B momentary 

C averted 

D detained 

Answer: A

 

Instructions For the following questions answer them individually 

Q.95 Select the most appropriate option to substitute the underlined segment in the given sentence. If no substitution is required, select No improvement. He made a desperate but also partial successful effort to change the ghastly topic. 

A No improvement 

B but partially successful effort 

C yet also partial successful effort 

D though part successful effort 

Answer: B 

 

Q.96 Given below are four jumbled sentences. Select the option that gives their correct order. 

A. But that is for an emotional reason. 

B. Learning our mother tongue is important and everyone should start there. 

C. This is because proficiency in the language can lead to better opportunities. 

D. Learning English, however, is important for practical reasons. 

A BADC 

B DACB 

C BDCA 

D DBAC 

Answer: A 

 

Q.97 Select the word which means the same as the group of words given. That which cannot be satisfied 

A insatiable 

B impossible 

C improbable 

D incredible 

Answer: A 

 

Q.98 Select the most appropriate synonym of the given word. CONCISE 

A detailed 

B brief 

C lengthy 

D complex 

Answer: B 

 

Instructions: Select the most appropriate meaning of the underlined idiom in the given sentence. 

Q.99 One must learn to prioritize in life. It never pays to put the cart before the horse. 

A do last things first 

B postpone till the last moment 

C perform many tasks simultaneously 

D do things spontaneously 

Answer: A 

 

Q.100 His success at such a young age speaks volumes for his talent. 

A publically announces 

B showers praise 

C boasts a lot 

D gives enough proof 

Answer: D 

×

Hello!

Click one of our representatives below to chat on WhatsApp or send us an email to info@vidhyarthidarpan.com

×